Download as pdf or txt
Download as pdf or txt
You are on page 1of 52

EXAM QUESTIONS OF “CHILD DISEASES 2 (FACULTY PEDIATRICS)” FOR STUDENTS

OF 4TH YEAR (VIII SEMESTER) 2023-2024 a.y.

1. A 5 year old boy was hospitalized with a preliminary diagnosis of chronic disease of lungs. He is ill
during 4 years. There is a constant wet cough, persistent moist rales in the lower lobe of the right lung.
Which of the following will prove diagnosis?
A. Biplane (two-dimensional) chest x-ray
B. Chest ultrasound
C. Bronchoscopy
D. Bronchography
E. Spirography

2. The girl of 6 months is ill during 10 days. The disease began acutely with high temperature and
catarrhal syndrome. During objective examination: paleness of skin and mucus, dyspnea, acrocyanosis,
cardiomegalia. During nonrheumatic carditis is surprised more often:
A. All covers of heart
B. Myocardium
C. Endocardium
D. Pericardium
E. Myocardium and pericardium

3. Child 5 yrs old was entered to the hospital with complaints of weakness, decline of appetite, shortness
of breath. Clinically it was revealed cardiomegaly, weakness of heart tones, arrhythmias, soft systolic
murmur on the apex. Acute nonrheumatic carditis was diagnosed. For acute nonrheumatic carditis is not
characteristic:
A. Tachycardia
B. Cardiomegalia
C. Dullness of heart tones
D. Pain in joints
E. Bradycardia

4. Child 8 yrs old was entered to the hospital with complaints of weakness, decline of appetite, shortness
of breath. Clinically it was revealed cardiomegaly, weakness of heart tones, arrhythmias, soft systolic
murmur on the apex. Acute nonrheumatic carditis was diagnosed. The forecast during acute nonrheumatic
carditis
A. Recovering
B. Death till 6 years
C. Death till 3 years
D. Death till 1 year
E. The illness during all life

5.Child 7 yrs old had acute nonrheumatic carditis 2 year ago. 1 year ago it was relapse of carditis. Mother
interrupted therapy after 1 month of treatment. Clinically it is revealed physical retardation, cardiac hump
and signs of cardiac failure.
a. Chronic nonrheumatic carditis is diagnosed. The forecast is:
b. Recovering
c. Cardiac insufficiency
d. Death till 3 years
e. Death till 1 year

6. Child is 3 month old. Clinically: cardiomegaly at birth, its rapid progress, formation of cardiac hump,
progressive left-heart cardiac insufficiency, refraction to the therapy. Fibroelastosis was diagnosed.
Cardinal in treatment of fibroelastosis is:
a. Heart transplantation
b. Surgical correction of infringements of hemodynamics
c. Catheterisation of heart cavities
d. Antibiotic therapy
e. Hormones therapy

7. Child 7 yrs old was entered to the hospital with complaints of weakness, decline of appetite,
shortness of breath. Clinically it was revealed cardiomegaly, weakness of heart tones, arrhythmias, soft
systolic murmur on the apex. Acute nonrheumatic carditis was diagnosed. Preparation of choise during
acute nonrheumatic carditis is:
A. Aspirin
B. Planquenil
C. Digoxin
D. Delagil
E. Lasix

7. Child 7 yrs old had acute nonrheumatic carditis 1 year ago. Mother interrupted therapy after 1 month
of treatmen Clinically it is observed periodic cardialgias, weak heart tones, tachyarrhythmia, functional
systolic murmur. Subacute nonrheumatic carditis is diagnosed. Preparation of choise is:
A. Aspirin
B. Prednisolone
C. Digoxin
D. Delagil
E. Indometacin

8. Child 9 yrs old was entered to the hospital with complaints of weakness, decline of appetite,
shortness of breath. Clinically it was revealed cardiomegaly, weakness of heart tones, arrhythmias, soft
systolic murmur on the apex. Acute nonrheumatic carditis was diagnosed. The daily dose of prednisolon
during acute nonrheumatic carditis is:
A. 0,5 – 1,5 mg/kg
B. 1,5 – 2,5 mg/kg
C. 2,5 – 3,5 mg/kg
D. 3,5 – 4,5 mg/kg
E. 5 mg/kg

9. Patient 7 years old entered clinic with complaints of edema under eyes, increase of body temperature
up to 37.8°С, discoloration of urination. 2 weeks ago he had tonsillitis. What test must be done first of
all?
A. Estimation of ASL-O
B. General analysis of urine
C. Ultrasound of abdomen
D. Bacteriologic study of pharynx smash
E. All transferred

10. At sick boy 10 years old with pyelonephritis urine was taken for analysis. The analysis of urine in
this case is not characterized by:
A. Proteinuria more than 1 g/l
B. Neutrophils leukocyturia
C. Plenty of cellular epithelium
D. Sometimes plenty of salts
E. The specific gravity of urine is normal

11. Рossible extragastric manifestations of H. pylori infection:


1. Epigastric pain and nausea
2. Vomiting
3. Dysphagia
4. Odynophagia
5. Anemia (refractory iron deficiency anemia)

12. School-age children and adolescents with Peptic ulcer disease more commonly present with:
1. Epigastric pain and nausea
2. Dysphagia
3. Odynophagia
4. Feeding difficulty
5. Crying episodes

13. Рossible extragastric manifestations of H. pylori infection:


1. Epigastric pain and nausea
2. Vomiting
3. Dysphagia
4. Odynophagia
5. Idiopathic thrombocytopenic purpura

14. Infants and younger children with Peptic ulcer disease usually present with:
1. Feeding difficulty, vomiting, crying episodes
2. Epigastric pain, nausea, dysphagia
3. Pain, localized fullness or mass
4. Hepatomegaly, splenomegaly, odynophagia
5. Jaundice, arthritis, perianal disease

15. Reliable noninvasive method of detecting H. pylori infection in patients who do not require
endoscopic evaluation:
1. Abdominal ultrasonography
2. Serology for IgG
3. 13 C-urea breath tests
4. pH monitoring
5. Demonstration of H. pylori in biopsy specimens

16. Reliable noninvasive method of detecting H. pylori infection in patients who do not require
endoscopic evaluation:
1. Abdominal ultrasonography
2. Serology for IgG
3. Colonoscopy
4. Stool antigen tests
5. Blood culture

17. What is the dose of omeprazole in children?


1. 50-100 mg/kg/day
2. 5-10 mg/kg/day
3. 1-2 g/day
4. 500 mg/day
5. 1.0-3.3 mg/kg/day

18. What drug creates a gel over the mucosal surface that protects from acid injury, has little systemic
absorption and, therefore, few adverse effects?
1. Sucralfate
2. Amoxicillin
3. Ursodeoxycholic acid
4. Levofloxacin
5. Esomeprazole

19. The vast majority of patients with periumbilical or epigastric pain or discomfort have:
1. Peptic ulcer
2. Cholecystitis
3. Cholelithiasis
4. Functional gastrointestinal disorder
5. Pancreatitis
20. The most common of the gallbladder anomalies (reported to be present in up to 18% of patients with
a functioning gallbladder), has no clinical significance and occurs in individuals of all ages and, more
commonly, in women
1. Bilobed gallbladder
2. Gallbladder diverticulum
3. Hourglass gallbladder
4. Septated gallbladder
5. Phrygian cap

21. Gallbladder that has two separate cavities , each drained by its own cystic duct and sometimes
supplied by its own cystic artery.
1. Bilobed gallbladder
2. Duplicated gallbladder
3. Phrygian cap
4. Gallbladder diverticulum
5. Hourglass gallbladder

22. What medications cause peptic ulcer disease via suppression of prostaglandin production?
1. Proton pump inhibitors
2. Histamine (H2) blockers
3. Cytoprotective agents
4. Nonsteroidal anti-inflammatory drugs
5. Antibiotics

23. Which of the following is the treatment of choice for peptic ulcer disease caused by H. pylori
infection?
1. H2 blockers
2. Erythromycin
3. PPI and erythromycin
4. PPI and azithromycin
5. PPI, amoxicillin, and clarithromycin

24. A 17-year-old male patient with a history of peptic ulcer disease comes to the clinic complaining of
pain in the abdomen and inability to digest. Upper endoscopy reveals diffuse inflammation of the stomach
lining. What is the MOST likely associated infection?
1. Streptococcus
2. Clostridium difficile
3. Helicobacter pylori
4. Lactobacillus
5. Enterobacter

25. Which of the following is the best next step after 8 weeks of proton pump inhibitor trial without
improvement of chronic epigastric discomfort?
1. H2 inhibitors
2. Continue proton pump inhibitors
3. Colonoscopy
4. Endoscopy with biopsy
5. Abdominal ultrasonography

26. Which of the following is common non-invasive test used to diagnose active Helicobacter pylori
infection?
1. Campylobacter-like organism (CLO) test
2. Serologic test
3. Urea breath test
4. Antinuclear antibody test
5. Urine analysis
27. Which of the following is common non-invasive test used to diagnose active Helicobacter pylori
infection?
1. Campylobacter-like organism (CLO) test
2. Stool antigen test
3. Serologic test
4. Antinuclear antibody test
5. Urine analysis

28. Which of the following is the best next step in the management of Functional dyspepsia with alarm
features?
1. Endoscopy
2. Proton pump inhibitors
3. Testing for Helicobacter pylori
4. Diet and lifestyle modification
5. H2 antagonists

29. What types of medication are considered for a client with peptic ulcer disease (PUD)?
1. Proton pump inhibitors, H2 receptor antagonists, antibiotics, mucosal protectants
2. H2 receptor antagonists, antibiotics, proton pump inhibitors, probiotics
3. H2 receptor antagonists, antibiotics, probiotics, antacids
4. H2 receptor antagonists, nonsteroidal anti-inflammatory drugs, proton pump inhibitors, mucosal
protectants
5. Nonsteroidal anti-inflammatory drugs, antibiotics, proton pump inhibitors, mucosal protectants,
probiotics

30. Which medications block gastric acid production to promote healing of the stomach lining?
1. Antacids
2. Cytoprotective agents
3. Nonsteroidal anti-inflammatory drugs
4. Antibiotics
5. Proton pump inhibitors

31. Which of the following is a side effect of omeprazole?


1. Increased risk of sepsis
2. Increased risk of enteric infection
3. Increased risk of developing hirsutism
4. Increased risk of anxiety
5. Increased risk of developing dermatitis nigricans

32. Which of the following is true about a duodenal ulcer?


1. Eating worsens the pain
2. Constipation is always present
3. Eating relieves the pain
4. Hematemesis is always present
5. NSAID use is the most common risk factor

33. Which of the following is the best diagnostic test for a duodenal ulcer?
1. Abdominal CT
2. Esophagogastroduodenoscopy (EGD)
3. Colonoscopy
4. Sigmoidoscopy
5. Abdominal ultrasonography

34. Procedure to evaluate the health and function of the gallbladder and biliary system?
1. Abdominal CT
2. Colonoscopy
3. Ultrasound of gallbladder
4. Gallbladder scintigraphy
5. Esophagogastroduodenoscopy (EGD)

35. What is the most common type of gallbladder stone?


1. Calcium stones
2. Struvite stones
3. Cholesterol stones
4. Uric acid stones
5. Oxalate stones

36. A nuclear scan may be used in the evaluation of the gallbladder for what purpose?
1. To check the function of the gallbladder
2. To detect the presence of very tiny stones
3. To accurately measure the size of the gallbladder
4. To detect emphysematous cholecystitis
5. To evaluate the presence of hemorrhagic cholecystitis

37. Which statement accurately describes cholelithiasis?


1. The presence of one or more gallstones in the gallbladder
2. The presence of one or more gallstones in the liver
3. The presence of one or more gallstones in the pancreas
4. The presence of one or more gallstones in the stomach
5. The presence of more than 5 gallstones in the liver

38. A 17-year-old girls presents to the emergency department with complaints of pain in her abdomen
that started an hour after eating lunch. Which of the following is the diagnostic modality of choice in this
patient?
1. CT scan with contrast
2. CT scan without contrast
3. Nuclear medicine scan
4. X-ray
5. Ultrasound

39. An obese teenager comes to the emergency room (ER) with episodes of vomiting and right upper
quadrant (RUQ) pain radiating to her right scapula. Which of the following imaging modality would be
most helpful to confirm acute cholecystitis?
1. CT abdomen
2. MRI abdomen
3. Plain x-ray of abdomen
4. Abdominal ultrasound
5. Esophagogastroduodenoscopy

40. In evaluating patients with biliary disease using an ultrasound, what classic finding suggests
cholelithiasis?
1. Echogenic focus with acoustic shadowing
2. Gallbladder wall thickening
3. Pericholecystic fluid
4. Common bile duct dilatation
5. Intrahepatic duct dilatation
41. A 7 yrs old child had elevation of temperature t° to 40°C in anamnesis. For the last 3 months he
presents fusiform swelling of fingers, ankle joints and knee joints, pain in the upper part of the sternum
and cervical part of the spinal column. What is the most probable diagnosis?
A. Septic arthritis
B. Juvenile rheumatoid arthritis
C. Toxic synovitis
D. Rheumatism
E. Osteoarthritis

42. A 14 year old female fell ill 3 months ago after cold exposure. She complained of pain in her hand
and knee joints, morning stiffness and fever up to 38oC. Interphalangeal, metacarpophalangeal and knee
joints are swollen, hot, with reduced ranges of motions; ESR of 45 mm/h, CRP (+++), RF (+). What
group of medicines would you recommend to the patient?
A. Sulfonamides
B. Tetracyclines
C. Fluorchinolones
D. Cephalosporines
E. Nonsteroid anti-inflammatory drugs

43. A 4 years old girl was hospitalized with complaints of pain and swelling in the right knee and an
ankle joints, morning stiffness, rapid fatigue, subfebrile temperature. She is ill for 4 months. Beginning of
illness she connects with ARI. The disease began with a knee violation. She received aspirin, but the
effect was absent. After 3 months the process has spread to the radiocarpal joint. Put a preliminary
diagnosis.
A. Rheumatic fever
B. Infectious-allergic arthritis
C. Systemic lupus erythematosis
D. Rheumatoid arthritis
E. Systemic scleroderma

44. A 6 years old girl with eye problems was consulted by ophthalmologist. Diagnosis of uveitis was
established. There are complaints of pain and swelling in the right knee and a ankle joints, rapid fatigue,
subfebrile temperature. She is ill for 4 months. Affection of the eyes is special for:
A. Infectious arthritis
B. Acute rheumatic lever
C. Juvenile rheumathoid arthritis
D. Overuse syndrome
E. Osteomyelitis

45. 3 yrs old girl has fever, rash, arthritis and signs of inflammation of internal organs. Systemic form
of the juvenile rheumatoid arthritis was suspected. It is characterized with:
A. Evanescent salmon-pink macular rash
B. Chronic pain and swelling of many joints in a symmetric fashion
C. Chronic asymmetric arthritis of large joints
D. Purpuric skin rash
E. Hemarthrosis

46. A 7 year old male presents to physician with the chief complaint of dark "cola colored" urine, facial
puffiness and abdominal pain for the past 2 days. 14 days ago he had a sore throat and fever. He has had
abdominal pain. His urine is dark. Urine analysis shows an increased specific gravity, RBCs are too
numerous to count. What is the most probable diagnosis?
A. Acute heart failure
B. Glomerulonephritis. Nephrotic Syndrome
C. Glomerulonephritis. Nephritic Syndrome
D. Acute infection of urinary tract
E. Hemolytic uremic syndrome

47. A 4 year old female has been limping with swelling of her right knee for several months. Physical
examination demonstrates swelling of her right knee, flexion contracture of 10 degrees and flexion to 120
degrees. Lab. data: WBC 8 g/l, with 45\% neutr., 47 lymphs\%, 8\% mon. Hgb 120 g/l. ESR 20mm/h.
Rheum. factor neg., ANA 1:640 speckled. What is the most probable diagnosis?
A. Juvenile Rheumatoid Arthritis
B. Rheumatism
C. SLE
D. Osteomyelitis
E. Infection Arthritis

48. The polycystic kidney disease was diagnosed at the boy of 3 years old. Mother complaints that the
boy has growth retardation, poor appetite, vomiting. Skin is pale, turgor of soft tissues is reduced, heart
rate - 120 per minute, harsh breathing at auscultation, abdomen is enlarged, soft. Biochemical tests – urea
– 14 mmol/l, creatinine – 0,130 mmol/l, protein – 58 g/l. Which condition has been developed?
A. Chronic kidney disease
C. Encephalopathy
D. Interstitial nephritis
E. Pyelonephritis
F. Acute heart failure

49. A 7-year-old boy developed small hypopigmented depressed scars after fingernail scratches; he has
been treated with nonsteroidal antiinflammatory drugs (NSAIDs) for a rheumatic disease 6 weeks ago. Of
the following, the NSAID that is most likely to cause such a unique skin reaction is
A. celecoxib
B. meloxicam
C. indomethacin
D. naproxen
E. ibuprofen

50. Hydroxychloroquine sulfate is an antimalarial drug important in the treatment of SLE and
dermatomyositis, particularly cutaneous manifestations of disease and to reduce lupus flares. Of the
following, the most important procedure that should be done routinely during the course of administration
is
A. gastric endoscopy
B. bone marrow examination
C. muscle biopsy
D. glucose-6-phophate dehydrogenase enzyme level assay
E. ophthalmological examination

51. Which of the following conditions is characterised by an abrupt loss of kidney function leading to a
rapid decline in the glomerular filtration rate (GFR), accumulation of waste products such as blood urea
nitrogen (BUN) and creatinine, and dysregulation of extracellular volume and electrolyte homeostasis?
1. Acute kidney injury
2. Chronic kidney disease
3. Pyelonephritis
4. Vesicoureteral reflux
5. Renal stones

52. Which of the following patients would be diagnosed with stage G3a chronic kidney disease?
1. Patient A: irreversible kidney damage; GFR 58 mL/min per 1.73 m2)
2. Patient B: irreversible kidney damage; GFR 95 mL/min per 1.73 m2)
3. Patient C: irreversible kidney damage; GFR 65 mL/min per 1.73 m2)
4. Patient D: irreversible kidney damage; GFR 44 mL/min per 1.73 m2)
5. Patient E: irreversible kidney damage; GFR 14 mL/min per 1.73 m2)

53. Which of the following patients would be diagnosed with stage G4 chronic kidney disease?
1. Patient B: irreversible kidney damage; GFR 95 mL/min per 1.73 m2)
2. Patient A: irreversible kidney damage; GFR 25 mL/min per 1.73 m2)
3. Patient C: irreversible kidney damage; GFR 65 mL/min per 1.73 m2)
4. Patient D: irreversible kidney damage; GFR 44 mL/min per 1.73 m2)
5. Patient E: irreversible kidney damage; GFR 14 mL/min per 1.73 m2)
54. Which of the following patients would be diagnosed with stage G5 chronic kidney disease?
1. Patient B: irreversible kidney damage; GFR 95 mL/min per 1.73 m2)
2. Patient C: irreversible kidney damage; GFR 65 mL/min per 1.73 m2)
3. Patient D: irreversible kidney damage; GFR 44 mL/min per 1.73 m2)
4. Patient A: irreversible kidney damage; GFR 5 mL/min per 1.73 m2)
5. Patient E: irreversible kidney damage; GFR 24 mL/min per 1.73 m2)

55. Which of the following is a diagnostic criterion for acute kidney injury (AKI)?
1. Persistent evidence of kidney damage or a decrease in function for at least 3 months
2. Hematuria with dysmorphic red blood cells
3. Increase in serum creatinine by ≥ 0.3 mg/dL from baseline within 48 hr
4. Heavy proteinuria (protein excretion greater than 3.5 g/24 hours)
5. Multiple cysts on renal imaging

56. Which of the following is a diagnostic criterion for acute kidney injury (AKI)?
1. Increase in serum creatinine to ≥ 1.5 times baseline within the prior 7 days
2. Persistent evidence of kidney damage or a decrease in function for at least 3 months
3. Hematuria with dysmorphic red blood cells
4. Heavy proteinuria (protein excretion greater than 3.5 g/24 hours)
5. Multiple cysts on renal imaging

57. Which of the following conditions is characterised by fever with or without abdominal pain, loin
pain together with a significant growth of bacteria (usually a single organism) on urine culture?
1. Acute cystitis
2. Chronic kidney disease
3. Acute pyelonephritis
4. Vesicoureteral reflux
5. Renal stones

58. What is vesicoureteral reflux grade 2?


1. Reflux only fills the ureter without dilation.
2. Reflux fills the ureter and the collecting system without dilation.
3. Reflux fills and mildly dilates the ureter and the collecting system.
4. Reflux fills and grossly dilates the ureter and the collecting system with blunting of the calices.
5. Massive reflux grossly dilates the collecting system. All the calices are blunted with a loss of
papillary impression, and intrarenal reflux may be present. There is significant ureteral dilation and
tortuosity.

59. What is vesicoureteral reflux grade 4?


1. Reflux only fills the ureter without dilation.
2. Reflux fills and grossly dilates the ureter and the collecting system with blunting of the calices.
3. Reflux fills the ureter and the collecting system without dilation.
4. Reflux fills and mildly dilates the ureter and the collecting system.
5. Massive reflux grossly dilates the collecting system. All the calices are blunted with a loss of
papillary impression, and intrarenal reflux may be present. There is significant ureteral dilation and
tortuosity.

60. What is vesicoureteral reflux grade 5?


1. Massive reflux grossly dilates the collecting system. All the calices are blunted with a loss of
papillary impression, and intrarenal reflux may be present. There is significant ureteral dilation and
tortuosity.
2. Reflux only fills the ureter without dilation.
3. Reflux fills the ureter and the collecting system without dilation.
4. Reflux fills and mildly dilates the ureter and the collecting system.
5. Reflux fills and grossly dilates the ureter and the collecting system with blunting of the calices.
61. What is pollakiuria (daytime frequency) in a patient with voiding/ bladder dysfunction?
1. Abnormally frequent small voids in a previously toilet-trained child with no evidence of polyuria or
urinary tract infection
2. Uncontrolled leakage of urine, which can be continuous or intermittent
3. The sudden and unexpected experience of an immediate need to void
4. Difficulty in the initiation of voiding
5. Observed behavior used to either postpone voiding or suppress urgency, including standing on tiptoe
and forcefully crossing the legs

62. Sulfasalazine is an effective drug in many rheumatic diseases; however, it is associated with severe
systemic hypersensitivity reaction. It is approved in all the following rheumatic disease except
A. polyarticular juvenile idiopathic arthritis (JIA)
B. oligoarticular JIA
C. systemic JIA
D. peripheral arthritis associated with juvenile ankylosing spondylitis
E. enthesitis associated with juvenile ankylosing spondylitis

63. You are meeting parents of a 14-year-old girl who has been treated with a monthly intravenous
cyclophosphamide for SLE-associated renal failure for the last 6 months; the mother is asking about the
long-term complications of this drug. All the following are long-term complications except
A. bone marrow suppression
B. bladder cancer
C. leukemia
D. lymphoma
E. infertility

64. Juvenile idiopathic arthritis has many subtypes. Arthritis must be present to make a diagnosis of any
subtype; involved joints oftenly have the following signs except
A. swelling
B. warm
C. erythema
D. limitation of movement
E. pain on movement

65. Oligoarthritis, the most common subtype of juvenile idiopathic arthritis, is defined as involving ≤4
joints within the 1st 6 mo of disease onset. Of the following, the most commonly affected joint is
A. hip
B. knee
C. elbow
D. wrist
E. metatarsal

66. Oligoarthritis predominantly affects the joints of the lower extremities rather than upper extremity
joints. Which of the following joints is never a presenting sign of oligoarthritis?
A. hip
B. knee
C. ankle
D. metatarsal
E. interphalangeal

67. A 4-year-old girl recently diagnosed with persistent oligoarticular juvenile idiopathic rheumatoid
arthritis (JIA); she has 3 involved joints including the right knee, right ankle, and left elbow; antinuclear
antigen (ANA) is significantly positive. Of the following, the MOST important step in the management of
this girl is
A. regular examination of locomotor system
B. periodic slit-lamp examination
C. periodic ANA monitoring
D. periodic erythrocyte sedimentation rate (ESR) monitoring
E. frequent C-reactive protein (CRP) monitoring

68. Antinuclear antigen (ANA) measurement test is useful in some rheumatologic diseases especially
with persistent oligoarticular juvenile idiopathic rheumatoid arthritis (JIA). All the following are more
likely to be correlated with ANA positivity except
A. anterior uveitis
B. younger age at disease onset
C. female sex
D. symmetrical arthritis
E. lower number of involved joints over time

69. Rheumatoid factor (RF)–positive polyarthritis is characterized by aggressive symmetric inflammation


of joints of both upper and lower extremities. Of the following, the extra articular manifestation that is
almost exclusively occur in RFpositive individuals is
A. fever
B. evanescent rash
C. uveitis
D. extensor surfaces nodules
E. pericarditis

70. A 5-year-old boy develops acute onset of high spiking fevers, lymphadenopathy,
hepatosplenomegaly, and purpura; he has been diagnosed with systemic Juvenile idiopathic arthritis (JIA)
since early childhood treated with antiinflammatory drugs. You suspect macrophage activating syndrome
(MAS). Of the following, the best test that distinguishes MAS from a flare of the primary disease is
A. leucopenia
B. falling ESR
C. hypofibrinogenemia
D. hypertriglyceridemia
E. evidence of hemophagocytosis in the bone marrow

71. Rheumatoid factor (RF)–positive polyarthritis usually accounts for < 10% of all juvenile idiopathic
arthritis (JIA) cases. The articular manifestation pattern ischaracterized by involvement of ≥5 joints in
both upper and lower extremities. Of the following, the least effective drug to induce remission for this
subtype of JIA is
A. NSAIDs
B. methotrexate
C. TNF-α antagonists
D. IL-1 inhibitors
E. IL-6 inhibitors

72. A 5-year-old boy has systemic juvenile idiopathic arthritis (sJIA) with systemic manifestations
including fever, hepatosplenomegaly, lymphadenopathy, and pericarditis. Of the following, the BEST
initial treatment for this boy is
A. NSAIDs
B. systemic glucocorticoids
C. TNF-α inhibitors
D. IL-1 antagonists
E. IL-6 antagonists

73. The child with polyarticular JIA often has a more prolonged course of active joint inflammation and
requires early and aggressive therapy. Of the following, the predictor that carries the worst prognosis is
A. old age at onset
B. rheumatoid factor (RF) seronegativity
C. absence of rheumatoid nodules
D. small numbers of affected joints
E. hip joint involvement
74. Spondyloarthritis may be overlapped clinically with other forms of juvenile idiopathic arthritis (JIA).
Clinical manifestations that help to distinguish spondyloarthritis from other forms of juvenile arthritis
include all the following except
A. arthritis of the sacroiliac joints
B. arthritis of the hips
C. enthesitis
D. asymptomatic uveitis
E. gastrointestinal inflammation

75. Systemic lupus erythematosus (SLE) is a chronic autoimmune disease characterized by multisystem
inflammation and the presence of circulating autoantibodies directed against self-antigens. It occurs in
both children and adults. All the following are features of childhood SLE except
A. it is has a more severe course
B. there is more widespread organ involvement
C. it usually presents before 5 year of age
D. fever, fatigue, hematologic abnormalities, and arthritis are common clinical manifestations
E. although renal disease is asymptomatic, it is oftenly present as nephrotic syndrome in adolescent age
group

76. Skin is a commonly involved organ by SLE. There are different cutaneous manifestations. Of the
following, the skin manifestation that is MOST suggestive of SLE in children is
A. malar rash
B. discoid rash
C. photosensitive rash
D. cutaneous vasculitis
E. livedo reticularis

77. Which of the following best defines pyelonephritis?


1. Infection of the entire urinary tract
2. Cystitis complicated by fever
3. Perirenal abscess formation
4. Urinary tract infection with obstruction
5. Infection of the kidneys/upper urinary tract

78. Which of the following is associated with uncomplicated cystitis?


1. Fever
2. Suprapubic tenderness
3. Costovertebral angle tenderness
4. Chills
5. Vomiting

79. Which of the following is associated with uncomplicated urinary tract infections?
1. Absence of symptoms
2. Viral etiology
3. Antibiotics are contraindicated
4. An elevated serum erythrocyte sedimentation rate, procalcitonin level, and C-reactive protein are
common
5. No structural abnormalities of the urinary tract

80. Which of the following describes the correct pathway of infection during the pathogenesis of
pyelonephritis?
1. Ureter, urethra, bladder, renal calyx, renal papilla
2. Urethra, bladder, ureter, renal papilla, renal calyx
3. Urethra, bladder, ureter, renal calyx, renal papilla
4. Ureter, bladder, renal calyx, ureter, renal papilla
5. Urethra, ureter, bladder, renal papilla, renal calyx
81. A 3-year-old is diagnosed with pyelonephritis. The bacteria that is growing in the culture is listed as
sensitive to ALL antibiotics. Which of the following antibiotics will FAIL to treat this patient?
1. Amoxicillin
2. Nitrofurantoin
3. Ceftriaxone
4. Cephalexin
5. Cefotaxime

82. Which of the following best defines vesicourethral reflux?


1. The retrograde passage of urine from the bladder into the upper urinary tract
2. The retrograde passage of urine from the ureter into the kidney pelvis
3. The retrograde passage of urine from the urethra into the bladder
4. The retrograde passage of urine from the renal pelvis into the nephron

83. Which of the following is required for a diagnosis of chronic kidney disease (CKD)?
1. Persistent evidence of kidney damage or a decrease in function for at least 3 months
2. Hematuria with dysmorphic red blood cells
3. Heavy proteinuria (protein excretion greater than 3.5 g/24 hours)
4. Oliguria (urine production less than 0.5 mL/m2/h)
5. Multiple cysts on renal imaging

84. Which of the following best describes the relationship between acute kidney injury (AKI) and
chronic kidney disease (CKD)?
1. Pre-existing CKD decreases the risk of AKI.
2. AKI and CKD are two separate, mutually exclusive entities.
3. AKI can slow the progression of pre-existing CKD.
4. Unresolved AKI may lead to CKD.
5. AKI decreases the risk of CKD.

85. Which of the following is true regarding the staging of CKD?


1. It is intended to identify those at greatest risk for progression and complications.
2. A low glomerular filtration rate measurement is required to diagnose CKD.
3. A patient diagnosed with stage 3 CKD has very little chance of disease progression.
4. Patients with stage 1 CKD should be treated with kidney replacement therapy.
5. Earlier stages of CKD are usually symptomatic.

86. What is the cause of anemia seen in patients with CKD?


1. Decreased red blood cell production
2. Excessive sequestration of red blood cells in the spleen
3. Vitamin deficiency
4. Deficient globin synthesis
5. Red cell membranopathy

87. Which of the following is involved in the pathogenesis of mineral and bone disorders seen in CKD
patients?
1. Increased glomerular filtration rate (GFR)
2. Decreased parathyroid hormone (PTH) activity
3. Increased calcitriol activity
4. Inadequate activation (hydroxylation) of vitamin D
5. Decreased phosphatonin activity

88. Which type of kidney replacement therapy has the most favorable outcome in CKD patients?
1. Home hemodialysis
2. Peritoneal dialysis
3. Kidney transplantation
4. In-center hemodialysis
5. Continuous hemodiafiltration

89. What is defined as an acute severe elevation in blood pressure (BP) with severe, life-threatening
symptoms and/or evidence of acute end-organ damage (eg, hypertensive encephalopathy, heart failure, or
renal injury)?
1. Hypertensive emergency
2. Hypertensive urgency
3. Acute nephritic syndrome
4. Nephrotic syndrome
5. Acute kidney injury

90. Which of the following would help distinguish between prerenal disease and acute tubular necrosis?
1. Fractional excretion of potassium, red blood cell casts, urine specific gravity
2. Fractional excretion of phosphorus, proteinuria, blood cell casts, urine osmolarity, urine specific
gravity
3. Urine sodium, fractional excretion of sodium (FENa), urine osmolarity, urine specific gravity
4. Urine sodium, white blood cells, eosinophils, urine osmolarity, proteinuria
5. Red blood cell casts, fractional excretion of phosphorus, urine osmolarity, BUN-to-sodium ratio

91. Which of the following patients would be diagnosed with stage II acute kidney injury?
1. Patient A: creatinine level increased from 1.2–2.5 mg/dL
2. Patient B: creatinine level increased from 3.7–4.4 mg/dL
3. Patient C: creatinine level increased from 1.0–1.2 mg/dL
4. Patient D: creatinine level increased from 1.5–2.7 mg/dL
5. Patient E: creatinine level increased from 0.5–0.8 mg/dL

92. A patient presents with fever, eosinophilia, elevated creatinine, and proteinuria after starting
penicillin for a Streptococcus infection and Ibuprofen because of fever. Which type of acute kidney injury
is most likely?
1. Prerenal
2. Postrenal
3. Renovascular disease
4. Intrinsic renal
5. Due to increased renal perfusion

93. A 5 yr old child with a recent pharyngitis presents with periorbital edema, hypertension, gross
hematuria, oliguria, and elevated creatinine. Which type of acute kidney injury (AKI) is most likely?
1. Prerenal AKI due to decreased renal perfusion
2. Intrinsic renal AKI related to acute postinfectious glomerulonephritis
3. Postrenal AKI related to anatomic obstructions to the lower urinary tract
4. Renovascular disease
5. Acute tubular necrosis

94. A neonate with a history of hydronephrosis seen on prenatal ultrasound studies presents with a
palpable bladder and elevated creatinine. Which type of acute kidney injury (AKI) is most likely?
1. Postrenal AKI related to congenital urinary tract obstruction, probably posterior urethral valves
2. Intrinsic renal AKI related to acute postinfectious glomerulonephritis
3. Prerenal AKI due to decreased renal perfusion
4. Renovascular disease
5. Acute tubular necrosis

95. When a patient presents with true volume depletion and prerenal acute kidney injury, which
treatment is most appropriate?
1. Diuretics
2. Albumin
3. Vasopressor support
4. Intravenous fluids
5. Salt and water restriction

96. Which of the following is a diagnostic criterion for acute kidney injury (AKI)?
1. Oliguria (urine volume ≤ 0.5 mL/kg/hr for 6 hr)
2. Persistent evidence of kidney damage or a decrease in function for at least 3 months
3. Hematuria with dysmorphic red blood cells
4. Heavy proteinuria (protein excretion greater than 3.5 g/24 hours)
5. Multiple cysts on renal imaging

97. Drug-induced lupus refers to the presence of SLE manifestations triggered by exposure to specific
medications, including antibiotics.Of the following, the drug that is most likely associated with drug-
induced lupus is
A. isoniazid
B. rifampin
C. nitrofurantoin
D. penicillin
E. tetracycline

98. Systemic lupus erythematosus (SLE) is often characterized by periods of flare and disease quiescence
or may follow a more smoldering disease course. All the following lab tests correlate with active disease
except
A. positive anti-nuclear antibody titer
B. positive anti–double-stranded DNA level
C. low serum complement level
D. high erythrocyte sedimentation rate
E. elevated C-reactive protein (CRP) value

99. A 12-year-old female adolescent has a recent diagnosis of SLE; she has a faint malar rash and mild
arthritis including both elbow and knee joints as well as the joints of small fingers.Of the following, the
MOST appropriate initial therapy for this patient is
A. systemic steroids
B. methotrexate
C. leflunomide
D. cyclophosphamide
E. hydroxychloroquine

100. Chronic urticaria may be caused by


a) Latex
b) Peanut
c) Iv immunoglobulin
d) Streptococcal pharyngitis
e) systemic lupus erythematosus

101. Anthracycline cardiotoxicity (doxorubicin [Adriamycin]) on rare occasion causes acute


inflammatory myocardial injury, but more classically results in dilated cardiomyopathy (DCM) and
occurs in up to 30% of patients given a cumulative dose of doxorubicin exceeding
a) 250 mg/m2
b) 350 mg/m2
c) 450 mg/m2
d) 550 mg/m2
e) 650 mg/m2

102. Restrictive cardiomyopathy (RCM) accounts for <5% of cardiomyopathy cases. RCM is
characterized by all the following except:
a) Normal ventricular chamber dimensions
b) Normal myocardial wall thickness
c) Preserved systolic function
d) Normal atrial chamber dimensions
e) High ventricular diastolic pressure

103. Diastolic dysfunction and normal systolic function are recognized features of
a) Dilated cardiomyopathy
b) Hypertrophic cardiomyopathy
c) Left ventricular noncompaction
d) Restrictive cardiomyopathy
e) Arrhythmogenic right ventricular cardiomyopathy

104. Renal childhood diseases responsible for chronic hypertension include


a) hemolytic-uremic syndrome
b) acute tubular necrosis
c) congenital dysplastic kidney
d) pyelonephritis
e) renal trauma

105. Define the wrong assertion about cardiomyopathy.


a) Cardiomyopathy is a chronic and progressive disease in which the heart muscle
b) abnormally enlarged, thickened and stiffened
c) Cardiomyopathy affects only children at any stage of their life
d) It is not gender, geographic, race or age specific
e) It is a rare disease when diagnosed in infants and young children

106. What is the main symptom of cardiomyopathy?


a) Arrhythmia
b) Fatigue
c) Difficulty breathing
d) Poor appetite
e) All transferred

107. What investigation can differentiate between hypertrophic, restrictive or dilated cardiomyopathy in
most cases?
a) 24-hour Holter monitor
b) ECG
c) Echocardiography
d) X-ray of chest
e) All that is mentioned

108. Child is 6 years old. He has frequent respiratory diseases. There is a poor tolerance to physical
activity, development of shortness of breath, sometimes accompanied by attacks of breathlessness.
Syncope often appears at an exercise, heart failure signs are present. Put the most probable diagnosis.
a) Idiopathic pulmonary hypertension
b) Idiopathic hemosiderosis
c) Idiopathic fibroalveolitis
d) Congenital carditis
e) Cardiomyopathy

109. Acute non-rheumatic carditis in young children mainly is caused by:


a) Bacteria
b) Toxins
c) Allergic conditions
d) Fungi
e) Viruses

110. Acute non-rheumatic carditis is characterized by:


a) Decreased heart borders
b) Elevated blood pressure
c) Increased heart sounds
d) Everything is correct
e) Cardiac rhythm impairment

111. Acute non-rheumatic carditis is characterized by:


a) Organic systolic murmur
b) Elevated blood pressure
c) Increased heart sounds
d) Rough systolic murmur on the apex
e) Functional systolic murmur

112. The 3rd stage of the heart failure in non-rheumatic carditis is characterized by
a) Pneumonia
b) Meningitis
c) Pulmonary edema
d) Splenomegaly
e) Hepatitis

113. The 1 st of the heart failure in non-rheumatic carditis is characterized by:


a) Swellings on the legs
b) Anasarca
c) Dry cough
d) Cardiac asthma
e) Dyspnea on physical load

114. Identify the criterion of carditis.


a) Damage of myocardium and endocardium
b) Cardialgias
c) Damage of epicardium
d) Only pericarditis
e) Damage of myocardium

115. Identify the typical symptom for myocarditis.


a) Heart borders narrowed
b) Strengthening of the heart tones
c) Syndrome Wolff–Parkinson–White syndrome
d) Heart borders dilatation
e) Heart borders hypertrophy

116. Identify the typical symptom for myocardiosclerosis.


a) Low blood pressure
b) High blood pressure
c) Bradypnea
d) Tachypnea
e) Arrhythmia

117. Identify the typical symptom for rheumatic arthritis.


a) Morning stiffness
b) Permanent course
c) Symptoms subside within 2 weeks
d) Pale skin over joints
e) All transferred

118. How long should be present arthritis in patient to diagnose JRA (according WHO criteria)?
a) More than 1 month
b) More than 2 months
c) More than 3 months
d) More than 6 months
e) More than 1 year

119. Choose the most typical sign of rheumatoid arthritis in children in difference from adults:
a) Symmetrical involvement of small joints
b) Rheumatoid nodules
c) Damage of neck and mandible joints
d) Resistance to the treatment
e) Rare mono- or pauciarticular types development

120. For polyarticilar form of JRA typical is damage of more than:


a) 1 joint
b) 3 joints
c) 5 joints
d) 10 joints
e) 4 joints

121. Rheumatoid arthritis is characterized by:


a. No changes in the bones and joint surfaces
b. Hemarthrosis availability
c. Narrowing of joint space
d. The absence of effusion in the joint cavity
e. Expansion of joint space

122. What means “pannus" in rheumatoid arthritis?


a. Edema and hyperemia around the affected joint
b. The form of joint deformation
c. Effusion in the joint cavity
d. Microvilli proliferation of the synovial membrane
e. The form of rheumatoid spine lesions

123. Which of the following is typical rheumatoid arthritis clinic in children (unlike adults)?
a. Frequent lesion of small hand joints
b. Deformation of joints
c. Frequent injury of the cervical spine
d. The symmetry of joint damage
e. Less developed of mono- and pauciarticular form

124. The lesion of the cervical spine in JRA usually occurs at the level of:
a. 1st vertebrae
b. 2-3 vertebrae
c. 5-6 vertebrae
d. 4th vertebrae
e. 7th vertebrae

125. Which of the following is the common cause of dilated cardiomyopathy?


1. Bacterial infections
2. Myocardial infarctions
3. Viral myocarditis
4. Autoimmune conditions
5. Tumors

126. Which of the following markers is used to diagnose myocardial cell injury?
1. Alanine aminotransferase
2. Myosin
3. Actin
4. Troponin
5. Hemoglobin

127. Which of the following tests is the gold standard for the diagnosis of myocarditis?
1. Scintigraphy
2. Endomyocardial biopsy
3. Electrocardiogram
4. Chest X-ray
5. Echocardiography

128. What is defined as a myocardial disorder in which the heart muscle is structurally and functionally
abnormal in the absence of coronary artery disease, hypertension, valvular disease, and congenital heart
disease sufficient to explain the observed myocardial abnormality?
1. Rheumatic heart disease
2. Acute rheumatic fever
3. Kawasaki disease
4. Mitral valve prolapse
5. Cardiomyopathy

129. What is characterized by cardiac dysfunction in which left ventricle relaxation and filling is
abnormal and is accompanied by elevated filling pressures
1. Systolic dysfunction
2. Diastolic dysfunction
3. Ventricular dilation
4. Ventricular outflow obstruction
5. Communication between the systemic and pulmonary sides

130. In individuals with a genetic predisposition to autoimmunity, a viral infection may initiate a chronic
autoimmune myocarditis leading to
1. Ventricular septal defect
2. Dilated cardiomyopathy
3. Atrial septal defect
4. Takotsubo cardiomyopathy (“broken heart syndrome”)
5. Rheumatic heart disease

131. What is a typical clinical manifestation of dilated cardiomyopathy?


1. Acute coronary syndrome
2. Fever
3. Heart failure
4. Erythema marginatum
5. Chorea

132. What may be considered for select patients with dilated cardiomyopathy and a high risk of sudden
cardiac arrest?
1. Implantable cardiac defibrillator
2. Angiotensin-converting enzyme (ACE) inhibitors
3. Angiotensin receptor blockers (ARBs)
4. Corticosteroids
5. Digoxin

133. Define the persistent oligoarthritis (a subtype of juvenile idiopathic arthritis).


1. Arthritis affecting 1-4 joints during 1st 6 months of disease and ≤4 joints throughout the disease
course
2. Arthritis affecting 1-4 joints during 1st 6 months of disease and >4 joints throughout the disease
course (after 1st 6 mo of disease)
3. Arthritis affecting ≥ 5 joints during 1st 6 months of disease and ≥5 joints throughout the disease
course
4. Arthritis affecting 1 joint during 1st 6 months of disease and ≤2 joints throughout the disease course
5. Arthritis affecting 2 joint during 1st 6 months of disease and ≤3 joints throughout the disease course

134. Define the extended oligoarthritis (a subtype of juvenile idiopathic arthritis).


1. Arthritis affecting 1-4 joints during 1st 6 months of disease and ≤4 joints throughout the disease
course
2. Arthritis affecting 1-4 joints during 1st 6 months of disease and >4 joints throughout the disease
course (after 1st 6 mo of disease)
3. Arthritis affecting ≥ 6 joints during 1st 6 months of disease and ≥7 joints throughout the disease
course
4. Arthritis affecting 1 joint during 1st 6 months of disease and ≤2 joints throughout the disease course
5. Arthritis affecting 2 joint during 1st 6 months of disease and ≤3 joints throughout the disease course

135. What laboratory tests are necessary to diagnose juvenile idiopathic arthritis?
1. Antinuclear antibodies (ANA)
2. Rheumatoid factor (RF)
3. Juvenile idiopathic arthritis is a clinical diagnosis without any diagnostic laboratory tests
4. Complete blood count (CBC)
5. Procalcitonin and C-reactive protein level

136. Which of the following conditions is characterised by arthritis in ≥1 joint with, or preceded by, fever
of at least 2 weeks in duration that is documented to be daily (quotidian) for at least 3 days and
accompanied by ≥1 of the following: evanescent (nonfixed) erythematous rash; generalized lymph node
enlargement; hepatomegaly or splenomegaly or both; serositis?
1. Oligoarthritis
2. Polyarthritis (RF negative)
3. Polyarthritis (RF positive)
4. Psoriatic arthritis
5. Systemic juvenile idiopathic arthritis

137. Which of the following conditions is characterised by arthritis affecting 1-4 joints during 1st 6
months of disease?
1. Systemic juvenile idiopathic arthritis
2. Oligoarthritis
3. Polyarthritis (RF negative)
4. Polyarthritis (RF positive)
5. Psoriatic arthritis

138. Which of the following conditions is characterised by arthritis affecting ≥5 joints during 1st 6
months of disease; a test for RF is negative?
1. Systemic juvenile idiopathic arthritis
2. Oligoarthritis
3. Polyarthritis (RF positive)
4. Polyarthritis (RF negative)
5. Psoriatic arthritis

139. Which of the following skin manifestations is associated with systemic lupus erythematosus?
1. Heliotrope rash
2. Photosensitivity
3. Rhinophyma
4. Necrolytic migratory erythema
5. Circinate balanitis

140. Which of the following hematologic pathologies is associated with systemic lupus erythematosus?
1. Leukocytosis, polycythemia, thrombocytosis
2. Isolated thrombocytosis
3. Isolated polycythemia
4. Leukopenia, anemia, thrombocytopenia
5. Isolated leukocytosis

141. Which of the following drugs should be added to the management of a systemic lupus erythematosus
(SLE) patient with positive antiphospholipid antibodies?
1. NSAIDs
2. Rituximab
3. Daily aspirin
4. Methotrexate
5. Prednisone

142. Of the 12 clinical features listed in the systemic lupus erythematosus (SLE) diagnostic criteria (the
American College of Rheumatology (ACR) criteria), how many need to be present to make a diagnosis of
systemic lupus?
1. 4
2. 5
3. 3
4. 6
5. 7

143. A 14-year-old presents because of an erythematous rash over the nose and cheeks. She also
complains of skin rash over the face and hands following exposure to sunlight. She has bilateral joint pain
and swelling in her hands and knees. Urine examination shows hematuria and proteinuria. Serological
testing shows low C3 and C4 with a positive antinuclear antibody test. Which of the following is the most
likely diagnosis in this patient?
1. Systemic lupus erythematosus
2. Poststreptococcal glomerulonephritis
3. Hemolytic uremic syndrome
4. Henoch-Schonlein purpura
5. IgA nephritis

144. What laboratory results can be detected in a child with acute post-streptococcal glomerulonephritis?
1. A urine sample positive for proteinuria and hematuria; an increased creatinine level; and a decreased
glomerular filtration rate (GFR)
2. A urine sample positive for glucose and hematuria; a decreased blood urea nitrogen (BUN); and a
normal glomerular filtration rate (GFR)
3. A urine sample negative for proteinuria and hematuria; a decreased blood urea nitrogen (BUN); a
decreased creatinine level; and a normal glomerular filtration rate (GFR)
4. A urine sample positive for increased sodium, potassium, and glucose; and an increased glomerular
filtration rate (GFR)
5. A urine sample positive for proteinuria and hematuria; a decreased blood urea nitrogen (BUN); a
decreased creatinine level; and a normal glomerular filtration rate (GFR)

145. What are the signs and symptoms of acute poststreptococcal glomerulonephritis?
1. Edema, hypertension, oliguria, hematuria and proteinuria
2. Edema, hypertension, polyuria, hematuria and proteinuria
3. Edema, hypotension, oliguria, hematuria and glucosuria
4. Edema, hypertension, crystalluria, hematuria and proteinuria
5. Edema, hypotension, oliguria, pyuria and proteinuria

146. Which of the following is associated with post-streptococcal glomerulonephritis?


1. Elevated antistreptolysin O (ASO) titers
2. Elevated C3, C4, and CH50 complement levels
3. Low blood pressure
4. Protein-to-creatinine ratio of <0.2
5. Hypotension
147. Which client is most at risk of developing acute streptococcal glomerulonephritis?
1. A 10-year-old child with the mumps
2. A 15-year-old girl with influenza
3. A 6-year-old child with impetigo
4. An 8-year-old boy with the chickenpox
5. A 17-year-old girl with the systemic lupus erythematosus

148. A 5-year-old boy was diagnosed with a streptococcal infection in the throat. The child was sent home
and began taking antibiotics. During health teaching, the boy's mother was told to monitor the child's
urine output because the boy was at risk of developing acute streptococcal glomerulonephritis. What is
the time frame for developing this complication of poststreptococcal throat infection?
1. 1–2 weeks
2. 2–3 weeks
3. 3–4 weeks
4. 4–5 weeks
5. 6-7 weeks

149. Which of the following is an indication for imaging in urinary tract infection?
1. Acute uncomplicated urinary tract infection
2. Acute pyelonephritis
3. Acute cystitis
4. Failure to improve with antibiotic therapy
5. Clinically relevant uropathogens in urine culture

150. In addition to the urinalysis results, which symptoms indicate that a urine culture test should be
done?
1. Diarrhea, vomiting
2. Vaginal or penile discharge
3. Edema, hypertension
4. Fever, increased urination urgency
5. Vomiting, headache
151. Systemic forms of JRA usually are characterized by:
a. Meningitis
b. Polyserositis
c. Nephritic syndrome
d. Abdominal syndrome
e. Pneumonia

152. What JRA eye damage is mainly manifested?


a. Conjunctivitis
b. Retinopathy
c. Iridocyclitis
d. Retinal degeneration
e. Cataracts

153. Child is 2 month old. Clinically: cardiomegaly at birth, its rapid progress, formation of cardiac
hump, progressive left-heart cardiac insufficiency, refraction to the therapy. Fibroelastosis was diagnosed.
It is possible to suspect fibroelastosis when:
a. The refraction to therapy tachycardia
b. Rough systolic murmur on apex
c. Bradiarythmia
d. Diastolic murmur on apex
e. Diastolic murmur on aorta

154. The girl of 6 months is ill during 10 days. The disease began acutely with high temperature and
catarrhal syndrome. During objective examination: paleness of skin and mucus, dyspnea, acrocyanosis,
cardiomegalia. During nonrheumatic carditis is surprised more often:
A. All covers of heart
B. Myocardium
C. Endocardium
D. Pericardium
E. Myocardium and pericardium

155. Child 8 yrs old was entered to the hospital with complaints of weakness, decline of appetite,
shortness of breath. Clinically it was revealed cardiomegaly, weakness of heart tones, arrhythmias, soft
systolic murmur on the apex. Acute nonrheumatic carditis was diagnosed. The forecast during acute
nonrheumatic carditis
a. Recovering
b. Death till 6 years
c. Death till 3 years
d. Death till 1 year
e. The illness during all life

156. Child 4yrs old complains of unpleasant feeling in heart, pain in abdomen. The boy in the maternity
hospital was diagnosed congenital heart defects (ventricular septum defect. Suddenly quickening of the
heartbeat at rest to 150-200 beats per minute was observed. What must parents do?
a. Call ambulance cardiologic help
b. Give Phenobarbital
c. Put child in a horizontal position
d. Put child in a horizontal position and call ambulance cardiologic help
e. Press on carotid sinus

157. Child 10yrs old with acute carditis complains of discomfort in the heart, constricting pain in the
chest, pain in the epigastric region, dizziness, vomiting. ECG pulse is 240 per 1 minute, complexes QRS
are not changed, wave P merges with T. Put the diagnosis.
a. Supraventricular paroxysmal tachycardia
b. Acute left heart failure with sinus tachycardia
c. Acute right heart failure with sinus tachycardia
d. Total heart failure with sinus tachycardia
e. Ventricular paroxysmal tachycardia

158. A 7 yrs old child had elevation of temperature t° to 40°C in anamnesis. For the last 3 months he
presents fusiform swelling of fingers, ankle joints and knee joints, pain in the upper part of the sternum
and cervical part of the spinal column. What is the most probable diagnosis?
a. Septic arthritis
b. Juvenile rheumatoid arthritis
c. Toxic synovitis
d. Rheumatism
e. Osteoarthritis

159. A 14 year old female fell ill 3 months ago after cold exposure. She complained of pain in her hand
and knee joints, morning stiffness and fever up to 38oc. Interphalangeal, metacarpophalangeal and knee
joints are swollen, hot, with reduced ranges of motions; ESR of 45 mm/h, CRP (+++), RF (+). What
group of medicines would you recommend to the patient?
a. Nonsteroid anti-inflammatory drugs
b. Tetracyclines
c. Fluorchinolones
d. Cephalosporines
e. Sulfonamides

160. A boy 5 years old was hospitalized because of pain in the neck, knees and decrease of movements
in these joints especially in the morning. Two weeks ago was ARI with tonsillitis. The disease have acute
onset: hyperthermia, hard movements of the head, pain and edema of joints. After anti-inflammatory
drugs the pain became less intensive, but decreasing of active movements still remained. During
examination: skin pallor, deformation of knee joints with decrease movements in them and neck. What
examination will help to put early diagnosis of JRA?
a. Coombs test
b. ESR, CRP
c. Biopsy of synovial membrane
d. Pheumatoid factor
e. US of joints

154. A child with allergic diathesis, 4 years old, was admitted to the hospital on the 5th day of the
disease. Acute respiratory viral infection with sub febrile body temperature, complaints of loss of
appetite, lethargy, anxiety and moans at night, irritability. There are bouts of cyanosis, shortness of
breath; rhonchi in the lungs, enlarged liver, decreased diuresis. Borders of the heart are dilated
moderately, weakened apical , tachycardia, muffled, bradyarrhythmia. Which of the following symptoms
is pathognomonic for acquired non-rheumatic carditis?
a) Migrated. ARVI
b) Flaccidity, moaning at night
c) Bradyarrhythmias
d) Expansion of the boundaries of the heart
e) Attacks of cyanosis

155. A child, 5 years old, was admitted to hospital on the 7th day of illness with complaints of an
increase in body temperature to 37.5 ° C, shortness of breath, cough, aggravated by changing the position
of the body. On examination, catarrhal changes were found in the throat, a little shortened percussion tone
in the back areas was observed above the lungs, fine rhonchi were heard, the respiration rate was 40 per 1
min. The left border of the heart is shifted by 2 cm to the left of the midclavicular line, the heart rate is
126 in 1 min. The liver is 2 cm below the costal arch. Daily diuresis is reduced, urination is normal . What
diagnosis can be established ?
a. Acute pneumonia
b. Acute bronchiolitis
c. Acute obstructive bronchitis
d. Rheumatic carditis
e. Acute non-rheumatic carditis

156. A girl, 9 years old, is in serious condition. Pale, respiratory rate 40 per min, pulse 130 per min,
heart sounds deaf, gallop rhythm. On the radiograph is determined cardiomegaly. On. Echocardiogram:
reduction of myocardial contractility, ventricular dilatation. On. ECG: impaired left ventricular
myocardial repolarization. What is the preliminary diagnosis?
a. Anomaly of the coronary arteries
b. Endocarditis
c. Fibroelastosis of the endocardium
d. Non-rheumatic carditis
e. Pericarditis

157. A 7 year old male presents to physician with the chief complaint of dark "cola colored" urine, facial
puffiness and abdominal pain for the past 2 days. 14 days ago he had a sore throat and fever. He has had
abdominal pain. His urine is dark. Urine analysis shows an increased specific gravity, rbcs are too
numerous to count. What is the most probable diagnosis?
a. Glomerulonephritis. Nephritic Syndrome
b. Glomerulonephritis. Nephrotic Syndrome
c. Acute heart failure
d. Acute infection of urinary tract
e. Hemolytic uremic syndrome

158. A 4 year old female has been limping with swelling of her right knee for several months. Physical
examination demonstrates swelling of her right knee, flexion contracture of 10 degrees and flexion to 120
degrees. Lab. Data: WBC 8 g/l, with 45% neutr., 47% lymphs, 8% mon. Hgb 120 g/l. ESR 20mm/h.
Rheum. factor negative.What is the most probable diagnosis?
a. Rheumatism
b. Lupus
c. Osteomyelitis
d. Juvenile rheumatoid arthritis
e. Infection Arthritis

159. A 3 year old child with weight defficiency suffers from permanent moist cough. In history there are
some pneumonias with obstruction. On examination: distended chest, dullness on percussion over the
lower parts of lungs. On auscultation: a great number of different rales. Level of sweat chloride is 80
mmol/l. What is the most probable diagnosis?
a. Pulmonary hypoplasia
b. Recurrent bronchitis
c. Bronchial asthma
d. Bronchiectasis
e. Mucoviscidosis (cystic fibrosis)

160. A child, 6 years old, was admitted to hospital on the 7th day of illness with complaints of an increase
in body temperature to 37.5 ° C, shortness of breath, cough, aggravated by changing the position of the
body. On examination, catarrhal changes were found in the throat, a little shortened percussion tone in the
back areas was observed above the lungs, fine rhonchi were heard, the respiration rate was 40 per 1 min.
The left border of the heart is shifted by 2 cm to the left of the midclavicular line, the heart rate is 126 in 1
min. The liver is 2 cm below the costal arch. Daily diuresis is reduced, urination is normal . What
diagnosis can be established ?
a) Acute pneumonia
b) Acute bronchiolitis
c) Acute obstructive bronchitis
d) Rheumatic carditis
e) Acute non-rheumatic carditis

161. Baby 7 years old, after an acute, non-rheumatic carditis of moderate severity six months ago; is in
the dispensary record of the cardio-rheumatologist. On examination, there are no complaints, with a
general satisfactory condition. Borders of the heart correspond to the age norm, rhythmic tones, slightly
muffled. Other systems without pathology. Which of the following cannot be carried out at this hour?
a) Complete blood count
b) Planned elective measles vaccination
c) Registration ECG
d) B radiography of the chest
e) The course of treatment cardiotrophic drugs

162. A boy, 10 years old, complains of fatigue, shortness of breath on exertion. 2 days before the
examination, the child suffered a sore throat. On examination, pale, bruised under the eyes. Identified the
expansion of the left borders of the heart, muffled. And the tone above the tip, there systolic murmur is
heard. Other systems and organs unchanged. Which of the clinical symptoms excludes the non-rheumatic
nature of heart failure?
a) Pallor, bruises under the eyes
b) Expansion of the left border of the heart
c) Transferred angina in history
d) Muffled. And tones over the top
e) Mitral valve insufficiency

163. Mother of a previously healthy 5 year old male complains of cough and wheeze. Boy had playing
with a small toy. During examination the right side of a chest show hyperresonance, diminished vocal
resonance and poor air entry. What is the most probable diagnosis?
a) Pneumonia
b) Asthma
c) Foreign body aspiration
d) Bronchitis
e) Bronchiolitis

164. A boy 5 years old was hospitalized because of pain in the neck, knees and decrease of movements in
these joints especially in the morning. Two weeks ago was ARI with tonsillitis. The disease have acute
onset: hyperthermia, hard movements of the head, pain and edema of joints. After anti-inflammatory
drugs the pain became less intensive, but decreasing of active movements still remained. During
examination: skin pallor, deformation of knee joints with decrease movements in them and neck. What
examination will help to put early diagnosis of JRA?
a) Coombs test
b) ESR, CRP
c) Biopsy of synovial membrane
d) Pheumatoid factor
e) US of joints

165. A 14 year old female fell ill 3 months ago after cold exposure. She complained of pain in her hand
and knee joints, morning stiffness and fever up to 38oC. Interphalangeal, metacarpophalangeal and knee
joints are swollen, hot, with reduced ranges of motions; ESR of 45 mm/h, CRP (+++), RF (+). What
group of medicines would you recommend to the patient?
a) Nonsteroid anti-inflammatory drugs
b) Tetracyclines
c) Fluorchinolones
d) Cephalosporines
e) Sulfonamides

166. Child 9yrs old with acute carditis complains of discomfort in the heart, constricting pain in the chest,
pain in the epigastric region, dizziness, vomiting. ECG pulse is 180 per 1 minute, complexes QRS are
changed (ventricular extrasystoles), wave P is absent. Put the diagnosis.
a) Acute left heart failure with sinus tachycardia
b) Acute right heart failure with sinus tachycardia
c) Total heart failure with sinus tachycardia
d) Ventricular paroxysmal tachycardia
e) Supraventricular paroxysmal tachycardia

167. Child 8 yrs old was entered to the hospital with complaints of weakness, decline of appetite,
shortness of breath. Clinically it was revealed cardiomegaly, weakness of heart tones, arrhythmias, soft
systolic murmur on the apex. Acute nonrheumatic carditis was diagnosed. The forecast during acute
nonrheumatic carditis
a) Recovering
b) Death till 6 years
c) Death till 3 years
d) Death till 1 year
e) The illness during all life

168. Violation of the bones growth in JRA occurs:


a) Throughout the skeleton
b) In the lower extremities
c) Mainly in the flat bones
d) In areas remote from the lesion
e) In areas that border to the affected joints

169. In neonates with total anomalous pulmonary venous return and marked pulmonary venous
obstruction, the chest x-ray demonstrates
a) Small heart
b) Enlarged heart
c) Prominent pulmonary artery
d) Prominent right ventricle
e) Subaortic stenosis
170. Diastolic dysfunction and normal systolic function are recognized features of
a) Dilated cardiomyopathy
b) Hypertrophic cardiomyopathy
c) Left ventricular noncompaction
d) Restrictive cardiomyopathy
e) Arrhythmogenic right ventricular cardiomyopathy

171. Renal childhood diseases responsible for chronic hypertension include


a) Hemolytic-uremic syndrome
b) Acute tubular necrosis
c) Congenital dysplastic kidney
d) Pyelonephritis
a) Renal trauma

172. The definitive treatment of primary congenital nephrotic syndrome is


a) Angiotensin-converting enzyme inhibitors
b) Prostaglandin synthesis inhibitors
c) Aggressive nutritional support
d) Bilateral nephrectomies
e) Renal transplantation

173. A 17-month-old girl presented with history of recurrent attacks of pneumonia since the age of 8
months. You suspect a predominant B-cell defect. Of the following, the BEST simple initial screening
test is
a) IgA measurement
b) IgG measurement
c) IgM measurement
d) IgE measurement
e) isohemagglutinins titer

174. Put a preliminary diagnosis. Patient 14yrs old complained of intense pain in the right lumbar region,
chills, accompanied by fever up to 39°C. The abdomen is soft, painful in the right area. Palpation of right
kidney is painful. In the blood: leukocytes 30.0 x109/L, ESR - 50 mm/hour. In urine an.: acid reaction,
leukocytes in the entire field of vision. According to the US - the left kidney is normal, the contours of the
right kidney are increased.
a) Right paranephritis
b) Acute right-sided pyelonephritis
c) Tuberculosis of the right kidney
d) Swelling of the right kidney
e) Polycystic kidney degeneration

175. Mother of a girl 7 years old complained of recurrent abdominal pain and skin rash, increased
sweating, decrease in urine output and concentrated character. Nocturia is noted. AP is 90/60mm Hg.
General urine analysis: the relative gravity of urine - 1028, protein - 0,04g/l, Leuc. - 9-10 in v/f, Eryth. –
changed 6-8 in v/f, casts - not detected, salts - oxalates large number. Set a preliminary diagnosis.
a) Dysmetabolic nephropathy
b) Acute glomerulonephritis with nephritic syndrome
c) Urinary infection
d) Tubulopathy
e) Acute renal failure

176. The polycystic kidney disease was diagnosed at the boy of 3 years old. Mother complaints that the
boy has growth retardation, poor appetite, vomiting. Skin is pale, turgor of soft tissues is reduced, heart
rate - 120 per minute, harsh breathing at auscultation, abdomen is enlarged, soft. Biochemical tests – urea
– 14 mmol/l, creatinine – 0,130 mmol/l, protein – 58 g/l. Which condition has been developed?
a) Chronic kidney diseases
b) Acute renal failure
c) Encephalopathy
d) Interstitial nephritis
e) Pyelonephritis

177. A 10-year-old boy has consistently noted for 2 days: swelling of the knee, then ankle joints,
temperature 380C. The left border of the heart is increased by 2 cm. Heart sounds are muffled. A week
ago suffered a sore throat Your preliminary diagnosis:
1. Postinfectious myocarditis
2. Acute rheumatic fever
3. Juvenile rheumatoid arthritis
4. Septic carditis
5. Acute renal failure

178. Non-obstructive form of hypertrophic cardiomyopathy:


A. Concentric myocardial hypertrophy;
B. Apical myocardial hypertrophy;
B. Hypertrophy of the interventricular septum;
D. Hypertrophy of the anterior wall of the left ventricle
E. Hypertrophy of the inferior wall of the left ventricle;

179. Small criteria for the diagnosis of infective endocarditis:


A. Oscillating formations on valve cusps or on trabeculae or artificial valve;
B. Intracardiac abscess;
B. Dehiscence of the artificial valve;
D. Newly formed valve failure;
E. Fixed or inactive formations on the valves;

180. Large criteria for the diagnosis of infective endocarditis (blood culture results):
A. Positive blood culture result in both samples taken at 5 min intervals;
B. Positive blood culture in three samples. taken with an interval between the first and the last 60
minutes;
B. Positive blood culture in two samples taken more than 12 hours apart;
D. Positive result (growth of green streptococcus) in one of two samples taken at intervals of more than
12 hours;
E. A positive result (growth of bovine streptococcus) in two of four, taken with an interval between the
first and fourth sample of 70 minutes;

181. Small criteria for the diagnosis of infective endocarditis (blood culture results):
A. Positive blood culture result in both samples taken at 5 min intervals;
B. Positive blood culture in three samples. taken with an interval between the first and the last 60
minutes;
B. Positive blood culture in two samples taken more than 12 hours apart;
D. Positive result (growth of green streptococcus) in one of two samples taken at intervals of more than
12 hours;
E. A positive result (growth of bovine streptococcus) in two of four, taken with an interval between the
first and fourth sample of 70 minutes;

182. Which of the proposed criteria is the most important for the diagnosis of nephrotic syndrome (NS)?
A. Edema
B. Serum albumin below 30 g / l
B. Daily proteinuria more than 3.5 g
D. Hypercholesterolemia
E. Hypercoagulation

183. Indicate a condition in which the appointment of steroids is absolutely contraindicated:


A. Jade with minimal changes
B. Active lupus nephritis
B. True sclerodermic kidney
D. Kidney damage in Wegener's granulomatosis
D. Mesangioproliferative nephritis

184. Which of the following forms of AKI is hypercatabolic?


a. Medicinal
b. Postrenal
c. Crash syndrome
d. Prerenalnaya
e. Hepato-renal syndrome

185. Specify a symptom not typical for AKI:


A. Acute onset
B. Oliguria
B. Reduced kidney size
D. Hyperkalemia
D. Collapse

186. Treatment of choice in many centers for patients with restrictive cardiomyopathy
1. Implantable cardioverter-defibrillator
2. Prostaglandins
3. Cardiac transplantation
4. Nonsteroidal anti-inflammatory drugs
5. Prednisolone

187. One of 4 components of Tetralogy of Fallot:


1. Atrial septal defect
2. Transposition of the great arteries
3. Coarctation of the aorta
4. Right ventricular hypertrophy
5. Mitral stenosis

188. One of 4 components of Tetralogy of Fallot:


1. Obstruction to right ventricular outflow (pulmonary stenosis)
2. Atrial septal defect
3. Transposition of the great arteries
4. Coarctation of the aorta
5. Mitral stenosis

189. One of 4 components of Tetralogy of Fallot:


1. Atrial septal defect
2. Transposition of the great arteries
3. Coarctation of the aorta
4. Mitral stenosis
5. Ventricular septal defect

190. One of 4 components of Tetralogy of Fallot:


1. Atrial septal defect
2. Dextroposition of the aorta
3. Transposition of the great arteries
4. Coarctation of the aorta
5. Mitral stenosis

191. Define the following is a major criterion used to diagnose rheumatic fever.
1. Erythema marginatum
2. Erythema multiforme
3. Fever
4. Sore throat
5. Prolonged P-R interval

192. Define the the following is a minor criterion used to diagnose rheumatic fever?
1. Prolonged P-R interval
2. Chorea
3. Erythema multiforme
4. Sore throat
5. Subcutaneous nodule

193. Which of the following is a minor criterion used to diagnose rheumatic fever?
1. Elevated acute-phase reactants (erythrocyte sedimentation rate; C-reactive protein)
2. Supporting evidence of antecedent group A streptococcal infection
3. Carditis
4. Polyarthritis
5. Subcutaneous nodules

194. A 12-year-old boy has sore throat, fever, tender and swollen anterior cervical nodes, and bilateral
tonsillar exudates for 2 days. He has no coryza, conjunctivitis, cough, hoarseness, anterior stomatitis,
discrete ulcerative lesions or vesicles, or diarrhea. What is the next best step in management?
1. Obtain a rapid streptococcal antigen test
2. Initiate antibiotics
3. Obtain blood cultures
4. Prescribe acetaminophen
5. Refer for hospitalization

195. A 7-year-old boy with no known drug allergies is brought into the primary care office for 3 days of
sore throat and fevers and a slight "tummy ache" starting today. His mom states that he has been refusing
most food and drink because he states it hurts to swallow, but he will take frozen ice and sips of cold
water or juice. On physical exam, he has moist mucous membranes and good skin turgor. He has swollen
tonsils with exudate, and tender swollen anterior cervical nodes. A "Rapid Strep" test is done in the office
and comes back positive. What is the next best step in management?
1. A 10-day course of erythromycin
2. Throat cultures and confirmation of diagnosis before prescribing antibiotics
3. A 5-day course of oral penicillin
4. A 10-day course of oral penicillin or amoxicillin
5. A single intramuscular injection of long acting penicillin followed by a 14 day course of oral
antibiotics

196. onditions leads to left to right shunting?


1. Pulmonary stenosis
2. Patent ductus arteriosus
3. Atherosclerotic heart disease
4. Coarctation of the aorta
5. Aortic stenosis

197. Which one of the listed conditions leads to left to right shunting?
1. Pulmonary stenosis
2. Atherosclerotic heart disease
3. Atrial septal defect
4. Coarctation of the aorta
5. Aortic stenosis

198. Classic continuous machinery murmur is typically heard in which of the following conditions?
1. Aortic stenosis
2. Patent ductus arteriosus
3. Ventricular septal defect
4. Coarctation of aorta
5. Atrial septal defect

199. Define of the following congenital heart lesions is most likely to require surgical attention in early
childhood.
1. Transposition of the great arteries leading to blue baby
2. Atrial septal defect
3. Patent ductus arteriosus
4. Pulmonic stenosis
5. Ventricular septal defect

200. Define of the following congenital heart lesions results in persistent blood flow directly from the
aorta into the pulmonary artery.
1. Patent ductus arteriosus
2. Atrial septal defect
3. Ventricular septal defect
4. Coarctation of the aorta
5. Pulmonic stenosis

201. Define of the following conditions is most likely to present with cyanosis?
1. Atrial septal defect
2. Ventricular septal defect
3. Coarctation of the aorta
4. Bicuspid aortic valve
5. Transposition of the great arteries

202. In coarctation of the aorta, what would be the expected quality of the pulses and blood pressure?
1. Notably decreased in vessels before point of coarctation
2. Notably decreased in vessels below the coarctation
3. Notably increased in vessels below the coarctation
4. Unaffected by the coarctation
5. Notably increased only in collateral circulation

203. Which of the following medications should be administered to keep the patent ductus arteriosus
open?
1. Indomethacin
2. Prednisolone
3. Montelukast
4. Prostaglandin E1
5. Atropine

204. What is the first medication you give a baby with suspected ductal dependent cyanotic heart disease?
1. Prostaglandins
2. Epinephrine drip
3. Bicarb infusion
4. Surfactant
5. Indomethacin

205. Which of the following is a common finding in aortic coarctation?


1. Hypertension
2. Left atrial hypertrophy
3. Patent ductus arteriosus
4. Right ventricular hypertrophy
5. Tachycardia
206. A 17-year-old student and football player is referred to your primary care office for evaluation of
high blood pressure. During an annual check-up, he was noted to have an arterial blood pressure of
155/90 mm Hg without presenting symptoms. On physical examination, his blood pressure is 152/94 mm
Hg, and his heart rate is 59 beats/min. On auscultation, a systolic ejection murmur is heard over the spine
between the shoulder blades. Of note is that his femoral pulses are not well felt. His laboratory test results
are normal; however, an ECG indicates left ventricular hypertrophy. Which of the following is the most
likely diagnosis?
1. Aortic stenosis
2. Coarctation of the aorta
3. Bicuspid aortic valve
4. Hypertrophic obstructive cardiomyopathy
5. Patent ductus arteriosus

207. Which of the following cardiac diseases is acyanotic congenital heart lesion causing a volume load?
1. Coarctation of the aorta
2. Tetralogy of Fallot
3. Atrial septal defect
4. Transposition of the great arteries
5. Aortic stenosis

208. Which of the following cardiac diseases is acyanotic congenital heart lesion causing a volume load?
1. Coarctation of the aorta
2. Tetralogy of Fallot
3. Transposition of the great arteries
4. Aortic stenosis
5. Ventricular septal defect

209. Which of the following cardiac diseases is acyanotic congenital heart lesion causing a volume load?
1. Patent ductus arteriosus
2. Coarctation of the aorta
3. Tetralogy of Fallot
4. Transposition of the great arteries
5. Aortic stenosis

210. Which of the following cardiac diseases is acyanotic congenital heart lesion causing a pressure load?
1. Coarctation of the aorta
2. Patent ductus arteriosus
3. Tetralogy of Fallot
4. Transposition of the great arteries
5. Aortic stenosis

211. Which of the following cardiac diseases is cyanotic congenital heart lesion where pulmonary blood
flow is decreased?
1. Coarctation of the aorta
2. Patent ductus arteriosus
3. Transposition of the great arteries
4. Tetralogy of Fallot
5. Ventricular septal defect

212. Which of the following cardiac diseases is cyanotic congenital heart lesion where pulmonary blood
flow is increased?
1. Transposition of the great arteries
2. Tetralogy of Fallot
3. Coarctation of the aorta
4. Patent ductus arteriosus
5. Ventricular septal defect
213. What cardiomyopathy is characterized predominantly by left ventricular dilation and decreased left
ventricular systolic function?
1. Hypertrophic cardiomyopathy
2. Restrictive cardiomyopathy
3. Dilated cardiomyopathy
4. Takotsubo cardiomyopathy
5. Viral myocarditis

214. What cardiomyopathy is demonstrating increased ventricular myocardial wall thickness and normal
or increased systolic function?
1. Dilated cardiomyopathy
2. Hypertrophic cardiomyopathy
3. Restrictive cardiomyopathy
4. Takotsubo cardiomyopathy
5. Viral myocarditis

215. What cardiomyopathy is occurring due to extreme psychological or physical stress (“broken heart
syndrome”)?
1. Dilated cardiomyopathy
2. Hypertrophic cardiomyopathy
3. Restrictive cardiomyopathy
4. Viral myocarditis
5. Takotsubo cardiomyopathy

216. Which of the following medications is most appropriate in a patient with heart failure due to
myocarditis?
1. Antiretrovirals
2. Angiotensin-converting enzyme inhibitors
3. Nonsteroidal anti-inflammatory drugs
4. Alcohol
5. Exercise

217. Which of the following is often a primary form of cardiomyopathy?


1. Hypertrophic cardiomyopathy
2. Viral infection
3. Uncontrolled thyrotoxicosis
4. Systemic lupus erythematosus
5. Hemochromatosis

218. Which of the following is the typical location of hypertrophy in hypertrophic cardiomyopathy?
1. Lateral wall of the left ventricle
2. Ventricular apex
3. Outflow tract of the right ventricle
4. Atria
5. Interventricular septum

219. A patient has pain on the right side of the abdomen radiating to the right shoulder. The serum
transaminases, alkaline phosphatase, and serum lipase are increased. Which location is the MOST likely
site of impaction of the stone?
1. The stone is impacted at the cystic duct.
2. The stone is impacted at the left hepatic bile duct.
3. The stone is present at the sphincter of Oddi.
4. The stone is impacted at the right hepatic duct.
5. The stone is impacted at the neck of the gallbladder
220. A 17-year-old girl presents with right upper quadrant pain. She has a BMI of 32 kg/m2. She
complains of feeling nauseated at times. What is the probable type of gallbladder stone causing the right
upper quadrant pain?
1. Mixed stones
2. Combined stones
3. Pigment stone
4. Uric acid stone
5. Pure cholesterol stone

221. What is defined as a motility disorder of the biliary tract that may cause biliary colic in children,
often in association with nausea and fatty food intolerance?
1. Biliary dyskinesia
2. Gastroduodenitis
3. Peptic ulcer disease
4. Cholecystitis
5. Cholelithiasis

222. What is defined as chronic relapsing multifactorial disease, manifested by the formation of ulcerative
defects (ulcers) in the stomach and / or duodenum (duodenum) with possible progression and
complications?
1. Gastroduodenitis
2. Biliary dyskinesia
3. Cholecystitis
4. Cholelithiasis
5. Peptic ulcer disease

223. What type of bronchitis is a common cause of chronic wet cough in preschool children with no
symptoms or signs of other specific causes, and resolution usually follows a 2-week course of an
appropriate oral antibiotic?
1. Protracted bacterial bronchitis
2. Acute bacterial bronchitis
3. Chronical bronchitis
4. Acute tracheobronchitis
5. Acute obstructive bronchitis

224. What respiratory rate might indicate that an infant has severe bronchiolitis?
1. < 50 breaths/min
2. 40–50 breaths/min
3. 30–40 breaths/min
4. 70 breaths/min
5. 20–30 breaths/min

225. What kind of supports can help infants manage viral bronchiolitis?
1. Antipyretics, nasal suction, hydration, supplemental oxygen
2. Antibiotics, nasal suction, hydration, stopping breastfeeding
3. Antipyretics, nasal suction, antibiotics, supplemental oxygen
4. Antibiotics, nasal suction, hydration, supplemental oxygen
5. Antipyretics, stopping breastfeeding, supplemental oxygen

226. What is the difference between bronchitis and bronchiolitis?


1. Bronchitis usually affects older children and adults, while bronchiolitis most often affects young
children under the age of two
2. Bronchiolitis is an inflammation of the lining of the bronchi. Bronchitis primarily affects the
bronchioles.
3. Both bronchitis and bronchiolitis are mostly caused by viral pathogens, but the main cause of
bronchiolitis is rhinovirus
4. Both bronchitis and bronchiolitis are mostly caused by viral pathogens, but the main cause of
bronchiolitis is adenovirus
5. Both bronchitis and bronchiolitis are mostly caused by viral pathogens, but the main cause of
bronchiolitis is parainfluenza virus
227. What is the age of a typical patient presenting with bronchiolitis?
1. Under 5 years of age
2. Over 3 years of age
3. Under 2 years of age
4. Between 1-2 years of age
5. Between 2-3 years of age

228. What is defined as a clinical syndrome of respiratory distress that occurs in children <2 years of age
and is characterized by upper respiratory symptoms (eg, rhinorrhea), followed by lower respiratory (eg,
small airway/bronchiole) infection with inflammation, which results in tachypnea, wheezing, crackles,
rales, rhonchi?
1. Bronchitis
2. Pneumonia
3. Bronchiolitis
4. Malacia of the larynx, trachea, and/or bronchi
5. Cystic fibrosis

229. Bronchiolitis is more common in


1. Boys, those exposed to secondhand tobacco smoke, those who have not been breastfed, those living
in crowded conditions.
2. Girls, those exposed to secondhand tobacco smoke, those who have not been breastfed, those living
in crowded conditions.
3. Boys, those exposed to secondhand tobacco smoke, those who have been breastfed, those living in
crowded conditions.
4. Girls, those exposed to secondhand tobacco smoke, those who have been breastfed, those living in
crowded conditions.
5. Girls, those exposed to secondhand tobacco smoke, those who have been breastfed, those not living
in crowded conditions.

230. Which of the following is a risk factor for hospitalization with severe bronchiolitis?
1. Cockroaches in the home
2. Breastfeeding
3. Being a single child
4. Being older than 6 months of age
5. Secondhand tobacco smoke

231. When a patient is diagnosed with acute bronchitis, which of the following investigations is can be
performed in order to rule out an accompanying pneumonia?
1. Chest X-ray
2. Bronchoscopy
3. High-resolution CT (HRCT) scan
4. Lung biopsy
5. MRI

232. 16-year-old male presents with a history of productive cough and low-grade fever. He was
previously healthy and does not suffer from any chronic ailments. He is diagnosed with acute bronchitis.
What is the next step in management?
1. Inhaled steroids
2. Inhaled bronchodilators
3. Supportive care only
4. Oral third generation cephalosporins
5. Oral neuraminidase inhibitors
233. Which factors would indicate a diagnosis of bronchitis?
1. History of acute onset of persistent cough for 1–3 weeks, no clinical signs of pneumonia (fever,
rales, tachypnea), chest X-ray to rule out pneumonia
2. History of acute onset of persistent cough for 1–3 weeks, no clinical signs of pneumonia (fever,
rales, tachypnea), an infiltrate on chest radiograph
3. History of acute onset of persistent cough for 1–3 weeks, no clinical signs of pneumonia (fever,
rales, tachypnea), a pleural effusion or empyema on chest radiograph
4. History of acute onset of persistent cough for 1–3 weeks, no clinical signs of pneumonia (fever,
rales, tachypnea), an elevated WBC count, in the range of 15,000-40,000/mm3
5. History of acute onset of persistent cough for 1–3 weeks, clinical signs of pneumonia (fever, rales,
tachypnea), positive blood culture

234. Which of the following is the preferred oral therapy for a patient with Streptococcus pneumoniae
with MICs for penicillin ≤ 2.0 µg/mL (susceptible to penicillin; step-down therapy or mild infection)?
1. Cefixime
2. Clindamycin
3. Amoxicillin
4. Levofloxacin
5. Linezolid

235. Which of the following best defines hospital-acquired pneumonia?


1. Pneumonia acquired 48 hours after a hospital admission unrelated to pneumonia
2. Pneumonia acquired in the community requiring hospitalization
3. Pneumonia developed by healthcare personnel
4. Pneumonia associated with foreign body aspiration
5. Pneumonia associated with medication

236. Which of the following symptoms represent atypical pneumonia caused by Mycoplasma
pneumoniae, Chlamydia pneumoniae?
1. Nonproductive cough, chills, localized chest pain, and myalgia
2. Productive cough, rhonchi, localized chest pain, and arthralgia
3. Nonproductive cough, rhonchi, wheezing, myalgia, and conjunctivitis
4. Nonproductive cough, chills, localized chest pain, rhonchi, and arthralgia
5. Productive cough, wheezing, chills, localized chest pain, and arthralgia

237. Which of the following is the most common cause of atypical pneumonia?
1. Streptococcus pneumoniae
2. Chlamydia trachomatis
3. Mycoplasma pneumoniae
4. Respiratory syncytial virus
5. Escherichia coli

238. What is the indication for doing a blood culture in patients with pneumonia?
1. Routine work-up for pneumonia
2. Suspicion of sepsis
3. Presence of hemoptysis
4. Negative chest x-ray findings
5. Non-productive cough

239. Which of the following is the antibiotic of choice for bacterial pneumonia in an otherwise healthy
child 1-4 years of age?
1. Azithromycin
2. Amoxicillin
3. Clindamycin
4. Moxifloxacin
5. Vancomycin
240. Which of the following is the bacterial pathogen most commonly found in cases of community-
acquired pneumonia?
1. Streptococcus pneumoniae
2. Group A Streptococci
3. Group B Streptococci
4. Staphylococcus aureus
5. Staphylococcus epidermidis

241. Which of the following drugs is added for the treatment of infants with pneumonia due to Chlamydia
trachomatis?
1. Ciprofloxacin
2. Clindamycin
3. Amoxicillin
4. Azithromycin
5. Moxifloxacin

242. What types of pathogens typically cause community-acquired pneumonia in children 0 to 18 years
old?
1. Streptococcus pneumoniae, Mycoplasma pneumoniae, Haemophilus influenzae, Chlamydia
pneumoniae
2. Mycoplasma pneumoniae, Haemophilus influenzae, Chlamydia pneumoniae, Staphylococcus aureus
3. Streptococcus pneumoniae, Haemophilus influenzae, Chlamydia pneumoniae, Chlamydophila
psittaci
4. Mycoplasma pneumoniae, Haemophilus influenzae, Chlamydia pneumoniae, Escherichia coli
5. Staphylococcus aureus, Haemophilus influenzae, Chlamydia pneumoniae, Measles

243. Which of the following test has a rapid response time and is reliable in providing the diagnosis of
mycoplasma pneumonia?
1. Mycoplasma antibody rise in titres
2. Polymerase chain reaction (PCR)
3. Sputum culture
4. Chest x-ray
5. Gram stain on sputum sample

244. Which neonatal infection is caused by Chlamydia trachomatis 2–3 weeks after birth?
1. Asthma
2. Emphysema
3. Pneumonia
4. Bronchitis
5. Bronchiectasis

245. What is the peak incidence of bronchiolitis hospitalization?


1. Between 12 and 14 months of age
2. Between 1 and 2 months of age
3. Between 2 and 6 years of age
4. Between 2 and 6 months of age
5. Between 12 and 14 years of age

246. What are the signs of bronchiolitis?


1. Persistent cough, tachypnea, and increased work of breathing (intercostal retractions, use of
accessory muscles, grunting, or nasal flaring)
2. Tachypnea, increased work of breathing (intercostal retractions, use of accessory muscles, grunting,
or nasal flaring), vomiting, anorexia, diarrhea
3. Tachycardia, persistent cough, and decreased work of breathing (intercostal retractions, use of
accessory muscles, grunting, or nasal flaring)
4. Abrupt onset of fever, restlessness, persistent cough, bradypnea, tachycardia
5. Bradypnea, vomiting, and increased work of breathing (intercostal retractions, use of accessory
muscles, grunting, or nasal flaring)

247. What demonstrate increased respiratory effort?


1. Persistent cough, tachypnea, bradycardia
2. Tachypnea, fever, restlessness
3. Persistent cough, fever, restlessness
4. Suprasternal, intercostal, and subcostal retractions
5. Persistent cough, bradypnea, tachycardia

248. The diagnosis of acute bronchiolitis is


1. Based on chest x-ray
2. Based on viral testing (polymerase chain reaction)
3. Based on мiral testing (rapid immunofluorescence)
4. Based on CT scan of the lungs
5. Clinical, when a previously healthy infant is presenting with a first episode of wheezing following a
period of upper respiratory symptoms

249. Severe bronchiolitis is indicated by any of the following:


1. Persistently increased respiratory effort, hypoxemia (SpO2 <95), apnea, acute respiratory failure
2. Persistently increased respiratory effort, hypoxemia (SpO2 <98), apnea, fever
3. Persistently cough, hypoxemia (SpO2 <95), tachypnea, acute respiratory failure
4. Persistently cough, hypoxemia (SpO2 <98), bradypnea, acute respiratory failure
5. Persistently increased respiratory effort, hypoxemia (SpO2 <99), tachypnea, acute respiratory failure

250. Find out the etiology of bronchitis in 90% of cases


1. Bacterial pathogens (Mycoplasma pneumoniae, Chlamydia pneumoniae, Bordetella pertussis)
2. Bacterial pathogens (Streptococcus pneumoniae, Moraxella catarrhalis, Haemophilus influenzae)
3. Gram-positive oropharyngeal flora
4. Respiratory viruses (parainfluenza virus, influenza virus, respiratory syncytial (RS) virus, rhinovirus,
adenovirus)
5. Gram-negative intestinal flora

251. Define the protracted bacterial bronchitis (PBB).


1. Common cause of chronic wet cough (daily cough that persists longer than 4 weeks) in preschool
children with no symptoms or signs of other specific causes, and resolution follows a 2-week course of an
appropriate oral antibiotic
2. Rare cause of chronic wet cough (daily cough that persists longer than 2 weeks) in preschool
children with no symptoms or signs of other specific causes, and resolution follows a 2-week course of an
appropriate oral antibiotic
3. Rare cause of chronic wet cough (daily cough that persists longer than 2 weeks) in newborns with no
symptoms or signs of other specific causes, and resolution follows a 2-week course of an appropriate oral
antibiotic
4. Common cause of chronic wet cough (daily cough that persists longer than 2 weeks) in newborns
with no symptoms or signs of other specific causes, and resolution follows a 4-week course of an
appropriate oral antibiotic
5. Common cause of chronic wet cough (daily cough that persists longer than 12 weeks) in preschool
children with no symptoms or signs of other specific causes, and resolution follows a 6-week course of an
appropriate oral antibiotic

252. Define the ventilator-associated pneumonia.


1. Pneumonia acquired 48 hours after endotracheal intubation or within 48 hours after extubation
2. Pneumonia acquired 48 hours after admission into the hospital for another reason
3. Pneumonia acquired 12 hours after endotracheal intubation or within 6 hours after extubation
4. Pneumonia in the setting of increased risk of aspiration such as poor gag reflex or a critically ill
status
5. Pneumonia caused by atypical organisms (e.g., Mycoplasma pneumoniae, Chlamydia pneumoniae)
and clinically characterized by milder symptoms with no lobar infiltrates on X-ray

253. Define the aspiration pneumonia.


1. Pneumonia acquired outside of the hospital setting
2. Pneumonia acquired 48 hours after admission into the hospital for another reason
3. Pneumonia acquired 48 hours after endotracheal intubation or within 48 hours after extubation
4. Pneumonia caused by atypical organisms (e.g., Mycoplasma pneumoniae, Chlamydia pneumoniae)
and clinically characterized by milder symptoms with no lobar infiltrates on X-ray
5. Pneumonia in the setting of increased risk of aspiration such as poor gag reflex or a critically ill
status

254. Define the atypical pneumonia.


1. Pneumonia in the setting of increased risk of aspiration such as poor gag reflex or a critically ill
status
2. Pneumonia caused by atypical organisms (e.g., Mycoplasma pneumoniae, Chlamydia pneumoniae)
and clinically characterized by milder symptoms with no lobar infiltrates on X-ray
3. Pneumonia acquired outside of the hospital setting
4. Pneumonia acquired 48 hours after admission into the hospital for another reason
5. Pneumonia acquired 48 hours after endotracheal intubation or within 48 hours after extubation

255. Identify the leading infectious killer of children worldwide.


1. Bronchiolitis
2. Bronchitis
3. Cystic fibrosis
4. Pneumonia
5. Bronchopulmonary dysplasia

256. What confirms the diagnosis of pneumonia?


1. The peripheral white blood cell (WBC) count
2. Erythrocyte sedimentation rate
3. Procalcitonin
4. C-reactive protein level
5. An infiltrate on chest radiograph (posteroanterior and lateral views)

257. How is recurrent pneumonia defined?


1. As 2 or more episodes in a single year or 3 or more episodes ever, with radiographic clearing
between occurrences
2. As 1 or more episodes in a single year or 2 episodes ever, with radiographic clearing between
occurrences
3. As 2 episodes in a single year or 3 or more episodes ever, without radiographic clearing between
occurrences
4. As 3 episodes in a single year or 4 or more episodes ever, without radiographic clearing between
occurrences
6. As 4 episodes in a single year or 5 or more episodes ever, without radiographic clearing between
occurrences

258. Define the optimal duration of antibiotic treatment for pneumonia.


1. Antibiotics should generally be continued until the patient has been afebrile for 72 hr, and the total
duration should not be less than 7-10 days (or 5 days if azithromycin is used)
2. Antibiotics should generally be continued until the patient has been afebrile for 12 hr, and the total
duration should not be less than 14-21 days (or 5 days if ceftriaxone is used)
3. Antibiotics should generally be continued until the patient has been afebrile for 24 hr, and the total
duration should not be less than 7-10 days (or 5 days if amoxicillin is used)
4. Antibiotics should generally be continued until the patient has been afebrile for 72 hr, and the total
duration should not be less than 14-21 days (or 5 days if amoxicillin is used)
5. Antibiotics should generally be continued until the patient has been afebrile for 24 hr, and the total
duration should not be less than 7-10 days (or 5 days if ceftriaxone is used)

259. An acute illness caused by an autoimmune response to infection with group A Streptococcus, leading
to a range of possible symptoms and signs affecting any or all of heart, joints, brain, skin and
subcutaneous tissues:
1. Acute rheumatic fever
2. Juvenile idiopathic arthritis
3. Cardiomyopathy
4. Systemic lupus erythematosus
5. Poststreptococcal glomerulonephritis

260. An acute illness that is diagnosed according to the Revised Jones Criteria and tends to recur with
subsequent group A streptococcal infections:
1. Juvenile idiopathic arthritis
2. Cardiomyopathy
3. Acute rheumatic fever
4. Systemic lupus erythematosus
5. Poststreptococcal glomerulonephritis

261. Active inflammation of the heart tissues, most importantly the mitral and/or the aortic valves, caused
by acute rheumatic fever:
1. Rheumatic heart disease
2. Rheumatic carditis
3. Cardiomyopathy
4. Systemic lupus erythematosus
5. Tetralogy of Fallot

262. The persistent damage to heart valves resulting in mitral and/or aortic regurgitation, or in long-
standing cases stenosis, that remains as a result of acute rheumatic fever.
1. Rheumatic carditis
2. Cardiomyopathy
3. Rheumatic heart disease
4. Systemic lupus erythematosus
5. Tetralogy of Fallot

263. A universal finding in rheumatic carditis:


1. Pericarditis
2. Myocarditis
3. Erythema marginatum
4. Chorea
5. Endocarditis

264. Once the diagnosis of acute rheumatic fever has been established, the patient should receive:
1. Penicillin or amoxicillin
2. Corticosteroids
3. Angiotensin receptor blockers (ARBs)
4. Angiotensin-converting enzyme (ACE) inhibitors
5. Furosemide

265. A single episode of acute rheumatic carditis often results in complete healing of the valvular lesions,
while repeated episodes, especially involving previously affected valves, result in
1. Pericarditis
2. Myocarditis
3. Rheumatic heart disease
4. Cardiomyopathy
5. Systemic lupus erythematosus
266. The diagnosis of Rheumatic heart disease is generally confirmed by:
1. Transthoracic echocardiography
2. Electrocardiography
3. Chest X-ray
4. Cardiac catheterization
5. Carotid duplex ultrasound

267. Paroxysmal hypercyanotic attacks (hypoxic, “blue,” or “tet” spells) are a problem during the 1st year
of life in patients with:
1. Hypertrophic cardiomyopathy
2. Tetralogy of Fallot
3. Patent ductus arteriosus
4. Coarctation of the aorta
5. Transposition of the great arteries

268.The patient, 15 years old, 5 years suffering from duodenal ulcer. After exercise felt severe pain in the
epigastric region and right upper quadrant. It was vomiting. There is a weak pulse, sharp pain from the
navel to the right, the disappearance of hepatic dullness during percussion. What caused condition that
requires emergency care?
a) Spontaneous pneumothorax.
b) Duodenal ulcer perforation.
c) Aggravation of kidney stones (renal colic).
d) Exacerbations of cholelithiasis
e) Ulcer bleeding

269. A girl, 12 years, complains of paroxysmal pain in the right upper quadrant, which increases during
exercise, after eating fatty foods, nausea, loss of appetite, headache, fatigue. Sick for 5 years. These
objective examination: skin pale tongue with yellow-gray stratification. Abdomen soft, painful in the right
upper quadrant, positive symptoms Ortner, Murphy, Kera, the liver acts from the edge of costal arch to 2
cm edge sharp. The tendency to constipation. These ultrasound: thickened wall and sealed intrahepatic bile
ducts and gall bladder. What is the main diagnosis you set sick?
a) Acute cholecystitis.
b) Chronic cholecystitis.
c) Chronic cholecystocholangitis.
d) Biliary dyskinesia hypokinetic by type
e) Biliary dyskinesia hypertensive type

270. In the 12-year-old boy there is aching pain in the epigastrium after 1,5 hours after eating and fasting,
periodic nausea, vomiting, heartburn. Recently, the child gets tired quickly, complaining of headache,
weakness. Chronic antral gastritis was diagnosed due to results of clinical and additional methods of
examinations. What clinical syndromes are the leading?
a) Pain, dyspepsial, asthenic-vegetative
b) Asthenic, pain, cephalgic
c) Pain, intoxication, asthenic
d) Dyspepsial, ossalgic, intoxication
e) Pain, intoxication, intoxication

271. The patient, 15 years old, 5 years suffering from duodenal ulcer. After exercise felt severe pain in the
epigastric region and right upper quadrant. It was vomiting. There is a weak pulse, sharp pain from the
navel to the right, the disappearance of hepatic dullness during percussion. What caused condition that
requires emergency care?
a) Duodenal ulcer perforation.
b) Spontaneous pneumothorax
c) Aggravation of kidney stones (renal colic)
d) Exacerbations of cholelithiasis
e) Ulcer bleeding
272. A 10 years old boy has acute glomerulonephritis during a mouth. He has edema. In urine: protein –
2,5 g/l, in the biochemical blood test: total protein – 48 g/l, cholesterol- 9,8 mmol|l. What from this medicine
must be appointed to the child in the complex of pathogenetical therapy?
a) Delagyl
b) Plaquenyl
c) Prednisolon
d) Heparin
e) Curantil

273. A 11-year-old boy is registered at a dispensary with a cardiorheumatologist with a diagnosis of


rheumatism, active phase, rheumacarditis with mitral valve damage. How long should secondary bicillin-
drug prevention of rheumatism be carried out?
a) for life
b) 3 years
c) 1 year
d) 18 years
e) 25 years

274. Pneumonia during auscultation is characterized by:


a) The presence of dry wheezing, which increases on exhalation, wheezing breathing
b) The presence of crepitation or small-bubbly wet wheezing, weakened or bronchial breathing
c) The presence of dry wheezing, scattered throughout the pulmonary fields, hard breathing
d) The presence of small bubbly wheezes at the height of inspiration, various wet and dry wheezes, which
change the character after coughing
e) The presence of wet and dry wheezing

275. Cabbage, oatmeal and buckwheat porridge, cheese, butter, baked potatoes, raisins, prunes and pumpkin
were included in the diet of a 9-year-old child. In what disease is such a diet justified?
a) Peptic ulcer disease
b) Acute pneumonia
c) Dysmetabolic nephropathy with oxalaturia
d) Rheumocarditis
e) Bronchial asthma

276. A 9-year-old child has been ill for three years. Complaints of abdominal pain, more in the right
hypochondrium, subfebrility. A positive symptom of Kera. In blood tests, an increase in ESR is noted. What
disease causes such a clinical picture:
a) Chronic enterocolitis
b) Chronic cholecystitis
c) Chronic duodenitis
d) Biliary dyskinesia
e) Acute intestinal infection

277. Identify the following diagnoses is most probable. A boy 12 years old admitted to the hospital with
intermittent high fever, allergic rash, pain and swelling in the knee and ankle joints, increase of
peripheral lymph nodes, liver and spleen. In blood test - leukocytes 27x109/l, ESR - 65mm/hour,
increased immunoglobulin M and G.
a) Sepsis
b) Systemic lupus erythematosus
c) Systemic juvenile rheumatoid arthritis
d) Rheumatc fever
e) Leukemia

278. Find the disease that led to the current state of the patient.13 years old girl was hospitalized with
straining pain in the left hypochondrium, which irradiates to the back. He notes nausea, decreased
appetite, weight loss, vomiting without relief, diarrhea. He has been ill for over 5 years. Exacerbation
has developed because of errors in the diet. Objective: t ° = 37,0°C, pulse rate 94 per minute, BP
125/75. Skin is pale, pain in the epigastrium, right and left hypochondrium. In the blood test: Leuk. 10.4
x 109/l, ESR 22 mm/hour.
a) Stomach ulcer
b) Chronic gastritis
c) Chronic cholecystitis
d) Chronic pancreatitis
e) Chronic enterocolitis

279. Indicate the drug which should be prescribed primarily for treatment. 6 years old boy complains of
an acute abdominal pain, which arises after mental loading, use of cold drinks, ice-cream. The
diagnosis: Dyskinesia of gallbladder, hypertonic type.
a) Spasmolitics and choleretics
b) Sedative and cholikinetics
c) Choleretics and cholikinetics
d) Antioxidants
e) Antibiotics

280. A 7-year-old child present with hypoalbuminemia, edema,hyperlipidemia and proteinuria.The edema
is in the periorbital region initially and eventually spreads to the rest of the body. The patient
is given steroid therapy and the disease goes away.What is a key morphological feature of the patient’s
disease?
a. Fusion of the foot processes
b. Destruction of the basement membrane
c. Destruction of the glomerulus
d. Hemosiderin laden macrophages in the kidney
e. Destruction of the loop of Henle function

281. Against the background of acute respiratory viral infections, an 11-year-old child developed a dry
cough. During auscultation, hard breathing, dry and single medium-bubbly diffuse wheezing on
inspiration, decreasing after coughing, is determined. Radiologically symmetrical enhancement of the
pulmonary pattern in the basal zones. In the general blood test – relative lymphocytosis. Specify the
probable diagnosis.
1.Acute obstructive bronchitis
2.Acute simple bronchitis
3.Bronchiolitis
4.Pneumonia
5.Recurrent bronchitis

282. A 2.5-year-old child was admitted to the hospital on the 3rd day of the disease. He became acutely ill,
the temperature was 39.2 C, weakness, coughing, refusal to eat, shortness of breath appeared from the 3rd
day, cough intensified. Upon admission: adynamic, pale, cyanosis of the nasolabial triangle, temperature -
38.5 C, respiratory rate - 52 per minute. Above the lungs - shortening of the sound in the right scapular
region. On the R-gram: homogeneous infiltration of segments 8-10 on the right. Make a diagnosis.
1. Obstructive bronchitis
2. Bronchitis
3. Interstitial pneumonia
4. Bronchiolitis
5. Segmental pneumonia

283. Which of the following is the best next step after 8 weeks of proton pump inhibitor trial without
improvement of chronic epigastric discomfort?
1) Colonoscopy
2) H2 inhibitors
3) Continue proton pump inhibitors
4) Endoscopy with biopsy
5) Abdominal ultrasonography
284.Which of the following medical conditions is the most common cause of dyspepsia?
1. Functional dyspepsia
2. Gastric cancer
3. Peptic ulcer disease
4. Pancreatitis
1. Esophageal cancer

285.What types of medication are considered for a client with peptic ulcer disease (PUD)?
1) H2 receptor antagonists, antibiotics, proton pump inhibitors, probiotics
2) H2 receptor antagonists, antibiotics, proton pump inhibitors, mucosal protectants
3) H2 receptor antagonists, antibiotics, probiotics, antacids
4) H2 receptor antagonists, nonsteroidal anti-inflammatory drugs, proton pump inhibitors, mucosal
protectants
5) Nonsteroidal anti-inflammatory drugs, antibiotics, proton pump inhibitors, mucosal protectants,
probiotics

286.Which of the following best describes the flow of bile from the liver to the gallbladder and then the
small intestines?
1. Right and left hepatic duct - Common hepatic duct - Cystic duct - Gallbladder - Cystic duct -
Common bile duct - Sphincter of Oddi
2. Right and left hepatic duct - Common bile duct - Cystic duct - Gallbladder - Cystic duct - Common
hepatic duct - Sphincter of Oddi
3. Sphincter of Oddi - Right and left hepatic duct - Common hepatic duct - Cystic duct - Gallbladder -
Cystic duct - Common bile duct
4. Right and left hepatic duct - Common bile duct - Gallbladder - Cystic duct - Common hepatic duct -
Sphincter of Oddi
5. Right and left hepatic duct - Gallbladder - Common bile duct - Cystic duct - Liver - Cystic duct -
Common hepatic duct - Sphincter of Oddi

287. An obese teenager comes to the ER with episodes of vomiting and RUQ pain radiating to her right
scapula. Which of the following imaging modality would be most helpful to confirm acute
cholecystitis?
1) CT abdomen
2) Abdominal ultrasound
3) MRI abdomen
4) Plain x-ray of abdomen
5) Esophagogastroduodenoscopy

288. A positive Murphy's sign is highly indicative of acute cholecystitis. Which of the following
statements demonstrates this maneuver?
1. Pain during inhalation as the examiner places his/her hand over the rt. upper quadrant
2. Pain during exhalation as the examiner places his/her hand over the rt. upper quadrant
3. Pain in rt. lower quadrant on palpation of the lt. lower quadrant
4. Pain during hyper-extension of the rt. hip joint
5. Pain on palpation over McBurney's point

289. Which hormone mediates the release of bile by causing gallbladder contraction when a person has a
lipid meal?
1. Chymotrypsin
2. Pepsin
3. Glucagon
4. Cholecystokinin
5. Trypsin

290. In evaluating patients with biliary disease using an ultrasound, what classic finding suggests
cholelithiasis?
6. Gallbladder wall thickening
7. Echogenic focus with acoustic shadowing
8. Pericholecystic fluid
9. Common bile duct dilatation
10. Intrahepatic duct dilatation

291. Which of the following is the treatment of choice for symptomatic cholelithiasis?
1. Cholecystectomy
2. Observation
3. Sphincterotomy
4. Ursodeoxycholic acid
5. Cholecystokinetic drug

292. What are risk factors that contribute to bronchiolitis? Select all that apply.
1. Being premature, attending daycare, being breastfed
2. Being premature, attending daycare, having heart or lung defects
3. Being premature, being the first child, having heart or lung defects
4. Being breastfed, attending daycare, having heart or lung defects
5. Older age, attending daycare, having heart or lung defects

293. How are bronchioles histologically different from bronchi?


1. Bronchiole walls do not have hyaline cartilage.
2. Bronchiole walls have hyaline cartilage.
3. Bronchiole walls do not have smooth muscle.
4. Bronchiole walls have smooth muscle.
5. Bronchioles are differentiated from bronchi based on size alone.

294. Which of the following options provides the correct order of lung structures from proximal to distal?
1. Terminal bronchioles, respiratory bronchioles, alveolar sacs, alveolar ducts
2. Respiratory bronchioles, terminal bronchioles, alveolar ducts, alveolar sacs
3. Respiratory bronchioles, terminal bronchioles, alveolar sacs, alveolar ducts
4. Terminal bronchioles, respiratory bronchioles, alveolar ducts, alveolar sacs
5. Terminal bronchioles, alveolar ducts, respiratory bronchioles, alveolar sacs

295. When a patient is diagnosed with acute bronchitis, which of the following investigations is usually
performed in order to rule out an accompanying pneumonia?
a. Chest X-ray
b. Bronchoscopy
c. HRCT-scan
d. Lung biopsy
e. MRI

296. As pneumonia progresses, what step occurs right before the consolidation of lung tissue?
1. Alveoli and respiratory bronchioles fill with serous exudate, blood cells, fibrin, and bacteria.
2. Alveolar edema and exudate formation occur.
3. An inflammatory response is initiated.
4. Infection to the lung (e.g., bacteria, viruses) occurs.
5. Bronchiolar obstruction with edema, mucus, and cellular debris

297. In patients with health care-associated pneumonia, which cephalosporin is initially included in the
antibiotic coverage regimen?
1. Ceftriaxone
2. Cefuroxime
3. Cefepime
4. Cefaclor
5. Cefazolin
298. Which of the following is the bacterial pathogen most commonly found in cases of community
acquired pneumonia?
a. Streptococcus pneumoniae
b. Group A Streptococci
c. Group B Streptococci
d. Staphylococcus aureus
e. Staphylococcus epidermidis
299. Which sign or symptom would suggest a primary viral pneumonia due to influenza over a typical
influenza illness?
1. Dyspnea
2. Consolidative sounds on auscultation
3. Fever
4. Productive cough
5. Significant myalgias

300. What types of pathogens typically cause pneumonia in children 0 to 18 years old?
1) Mycoplasma pneumoniae, Haemophilus influenzae, Chlamydia pneumoniae, Adenoviruses
2) Streptococcus pneumoniae, Haemophilus influenzae, Chlamydia pneumoniae, Influenza virus A and
B
3) Mycoplasma pneumoniae, Haemophilus influenzae, Chlamydia pneumoniae, Escherichia coli
4) Streptococcus pneumoniae, Mycoplasma pneumoniae, Haemophilus influenzae, Chlamydia
pneumoniae
5) Staphylococcus aureus, Haemophilus influenzae, Chlamydia pneumoniae, Influenza virus A and B

301. What factors should medical professionals consider while assessing work of breathing as part of the
pediatric assessment triangle (PAT)?
1. Nasal flaring, retractions, posturing, speech
2. Nasal flaring, retractions, cry, pulse
3. Nasal flaring, retractions, posturing, blood pressure
4. Nasal flaring, retractions, posturing, breath sounds
5. Nasal flaring, retractions, posturing, pulse

302. What assessments are part of a full respiratory examination?


1. Inspection, palpation, percussion, blood pressure
2. Inspection, palpation, percussion, auscultation
3. Inspection, pulse volume, percussion, auscultation
4. Inspection, palpation the liver, percussion, auscultation
5. Inspection, palpation, retraction, auscultation

303. Acute rheumatic fever most commonly occurs as a consequence of infection with of the following
pathogens?
1. Group A Streptococcus
2. Staphylococcus aureus
3. Listeria monocytogenes
4. Streptococcus pneumoniae
5. Neisseria meningitidis

304. Which of the following cardiac diseases is commonly caused by acute rheumatic fever?
1. Myocardial infarction
2. Infectious endocarditis
3. Mitral stenosis
4. Mitral valve prolapse
5. Pulmonic stenosis

305. In acute rheumatic fever, antibodies against group A streptococcus cross-react with antigens present
on which of the following structures of the heart?
1. Epicardium and myocardium
2. Myocardium and valves
3. Epicardium and endocardium
4. Epicardium and valves
5. Endothelium and epithelium

306. Which of the following is most likely the main indication for antibiotic treatment in children with
group A Streptococcus pharyngitis?
1. Reducing symptom severity
2. Prevention of post-streptococcal glomerulonephritis
3. Reducing symptom duration
4. Prevention of acute rheumatic fever
5. Prevention of spread to others

307. Which of the following is a major criterion used to diagnose rheumatic fever?
1. Carditis
2. Erythema multiforme
3. Sjogren’s syndrome
4. Sore throat
5. Hepatitis

308. Which of the following is a major criterion used to diagnose rheumatic fever?
1. Erythema multiforme
2. Polyarthritis
3. Sjogren’s syndrome
4. Sore throat
5. Prolonged P-R interval

309. Which of the following is a major criterion used to diagnose rheumatic fever?
1) Erythema marginatum
2) Erythema multiforme
3) Fever
4) Sore throat
5) Prolonged P-R interval

310. Which of the following is a major criterion used to diagnose rheumatic fever?
1. Erythema multiforme
2. Fever
3. Sore throat
4. Prolonged P-R interval
5. Subcutaneous nodules

311. Which of the following is a major criterion used to diagnose rheumatic fever?
1. Erythema multiforme
2. Fever
3. Sore throat
4. Chorea
5. Prolonged P-R interval

312. Which of the following is a minor criterion used to diagnose rheumatic fever?
1. Fever
2. Chorea
3. Erythema multiforme
4. Sore throat
5. Subcutaneous nodules
313. Which of the following is a minor criterion used to diagnose rheumatic fever?
1) Prolonged P-R interval
2) Chorea
3) Erythema multiforme
4) Sore throat
5) Subcutaneous nodules

314. Which of the following is a minor criterion used to diagnose rheumatic fever?
1) Elevated acute-phase reactants (erythrocyte sedimentation rate; C-reactive protein)
2) Supporting evidence of antecedent group A streptococcal infection
3) Carditis
4) Polyarthritis
5) Subcutaneous nodules

315. Rheumatic fever is a risk factor for which of the following?


1. Penicillin allergy
2. Valvular heart disease
3. Hypertrophic cardiomyopathy
4. Hypertension
5. Aneurysm

316. Which of the following valvular abnormalities is least likely to be a direct result of rheumatic heart
disease?
1. Pulmonary stenosis
2. Mitral stenosis
3. Mitral regurgitation
4. Aortic stenosis
5. Aortic regurgitation

317. Pericarditis is...


1. ...an inflammation of the heart's inner lining.
2. ...an inflammation of the heart muscle.
3. ...an inflammation of the valves of the heart.
4. ...an inflammation of the large blood vessels.
5. ...an inflammation of the sac surrounding the heart.

318. Which of the following describes the murmur of mitral stenosis?


1. Opening snap followed by a rumbling mitral diastolic murmur.
2. Mid-systolic click with a mid-diastolic rumble
3. Systolic crescendo-decrescendo murmur
4. Early diastolic decrescendo murmur
5. High-pitched holosystolic murmur

319. Which of the following is true regarding mitral stenosis?


1. Early-diastolic murmur, difficulty in closing
2. Diastolic murmur, difficulty in opening
3. Holosystolic murmur, difficulty in opening
4. Late systolic murmur, difficulty in opening
5. Crescendo-decrescendo, floppy valve

320. What are the cardinal features of a murmur related to mitral stenosis?
1. Located at the apex, heard during systole, causes increased pulse pressure
2. Located at the left sternal border, associated with an S4 sound and decreased pulse pressure
3. Located at the right upper sternal border, holosystolic, may radiate to the right carotid area
4. Located at the apex, heard during diastole, described as a low-pitched rumble
5. Located at the left sternal border, diastolic decrescendo murmur, sometimes associated with an S3
sound

321. Which part of the heart is often enlarged due to mitral stenosis?
1. Left atrium
2. Left ventricle
3. Right atrium
4. Aorta
5. Right ventricle

322. How would you differentiate mitral regurgitation from mitral stenosis in a gross pathologic specimen
of the heart?
1. The left atrium and left ventricle are enlarged in mitral regurgitation.
2. The left atrium is atrophied in mitral stenosis.
3. The left ventricle is hypertrophic in mitral stenosis.
4. The left atrium is atrophic in mitral regurgitation.
5. The interventricular septum is enlarged in mitral stenosis.

323. Which of the following is heard with mitral valve prolapse?


1. Mid-systolic click with or without a murmur
2. Diastolic murmur
3. Early systolic murmur
4. Murmur of mitral stenosis
5. Systolic murmur with an S3

324. Which one of the listed conditions leads to left to right shunting?
6. Pulmonary stenosis
7. Atherosclerotic heart disease
8. Patent ductus arteriosus
9. Coarctation of the aorta
10. Aortic stenosis

325. Which of the following is the most common type of atrial septal defect?
1. Sinus venosus
2. Ostium secundum
3. Ostium primum
4. Coronary sinus
5. Atrioventricular canal defect

326. Which of the following are the common causes of prerenal acute kidney injury?
1. Glomerulonephritis: postinfectious; lupus erythematosus; membranoproliferative
2. Hemolytic-uremic syndrome, acute tubular necrosis, cortical necrosis
3. Dehydration, sepsis, cardiac failure
4. Acute interstitial nephritis, tumor lysis syndrome, toxin and drugs
5. Posterior urethral valves, ureteropelvic junction obstruction, urolithiasis

327. Which of the following are the common causes of postrenal acute kidney injury?
1. Posterior urethral valves, ureteropelvic junction obstruction, urolithiasis
2. Dehydration, sepsis, cardiac failure
3. Glomerulonephritis: postinfectious; lupus erythematosus; membranoproliferative
4. Hemolytic-uremic syndrome, acute tubular necrosis, cortical necrosis
5. Acute interstitial nephritis, tumor lysis syndrome, toxin and drugs

328. Which of the following are the common causes of intrinsic renal acute kidney injury?
1. Posterior urethral valves, ureteropelvic junction obstruction, urolithiasis
2. Dehydration, sepsis, cardiac failure
3. Urethral strictures, Hemorrhagic cystitis, Neurogenic bladder
4. Glomerulonephritis: postinfectious; lupus erythematosus; membranoproliferative
5. Gastroenteritis, Hemorrhage, Burns, Anaphylaxis

329. What are the risk factors for pediatric acute kidney injury?
1. Critically ill patients, nephrotoxin use, comorbid conditions
2. All children < 5 years, fever, comorbid conditions
3. Critically ill adolescents, fever, comorbid conditions
4. Critically ill patients > 5 years, hypertension, fever
5. All children < 5 years, comorbid conditions, hypertension

330. A child of 2 months was delivered to the clinic on the 5th day from the onset of the disease. He
became acutely ill: the temperature rose to 38.2 C, an unproductive cough appeared, the child began to
refuse food. Upon admission to the clinic, the temperature is 38.5 C, BR = 70 per minute, shortness of
breath with a predominance of the expiratory component. The skin is pale, perioral cyanosis is
pronounced. There is a boxy sound above the lungs, the breathing is hard, on both sides there is an
abundance of small-bubbly wheezing on inhalation and exhalation. No focal changes were detected on the
X-ray of the chest organs. Your diagnosis:
1) Aspiration pneumonia
2) Bronchitis
3) Bronchial asthma
4) Foreign body
5) Bronchiolitis

331. Recurrent pneumonia is defined as:


A. 2 or more episodes in a single year, without radiographic clearing between occurrences
B. 2 or more episodes in a single year, with radiographic clearing between occurrences
C. 3 or more episodes in a single year, with radiographic clearing between occurrences
D. 3 or more episodes in a single year, without radiographic clearing between occurrences
E. 4 or more episodes in a single year

332. Which of the following conditions is characterised by arthritis affecting ≥5 joints during 1st 6 mo of
disease; a test for RF is negative?
a. Systemic juvenile idiopathic arthritis
b. Oligoarthritis
c. Polyarthritis (RF positive)
d. Psoriatic arthritis
e. Polyarthritis (RF negative)

333. Which of the following drugs used to treat lupus is a chimeric monoclonal antibody against the
protein CD20?
1. Rituximab
2. Hydroxychloroquine
3. Cyclophosphamide
4. Azathioprine
5. Mycophenolate

334. What are some laboratory results you may find in a child who has acute poststreptococcal
glomerulonephritis?
1) A urine sample positive for glucose and hematuria; a decreased blood urea nitrogen (BUN); and a
normal glomerular filtration rate (GFR)
2) A urine sample negative for proteinuria and hematuria; a decreased blood urea nitrogen (BUN); a
decreased creatinine level; and a normal glomerular filtration rate (GFR)
3) A urine sample positive for increased sodium, potassium, and glucose; and an increased glomerular
filtration rate (GFR)
4) A urine sample positive for proteinuria and hematuria; an increased creatinine level; and a decreased
glomerular filtration rate (GFR)
5) A urine sample positive for proteinuria and hematuria; a decreased blood urea nitrogen (BUN); a
decreased creatinine level; and a normal glomerular filtration rate (GFR)

335. A patient has a low C3 but a normal C4. What is the most likely cause of their hematuria?
1. Pyelonephritis
2. IgA nephropathy
3. Lupus nephritis
4. Membranoproliferative glomerulonephritis (MPGN)
5. Poststreptococcal glomerulonephritis (PSGN)

336. A 6-year-old boy was diagnosed with a streptococcal infection in the throat. The child was sent home
and began taking antibiotics. During health teaching, the boy's mother was told to monitor the child's
urine output because the boy was at risk of developing acute streptococcal glomerulonephritis. What is
the time frame for developing this complication of poststreptococcal throat infection?
1) 1–2 weeks
2) 2–3 weeks
3) 3–4 weeks
4) 4–5 weeks
5) 6-7 weeks

337. Which statement about poststreptococcal glomerulonephritis is correct?


1. It results from a viral infection of the throat or skin and tends to present 6 weeks after the infection.
2. Clients are less likely to experience hematuria with this condition.
3. Antibodies develop and deposit within the kidney, and they can cause inflammation.
4. Clients with this condition should consume a diet rich in sodium to prevent hyponatremia.
5. The body develops antibodies, which deposit in the ureters, causing urine to back up into the kidney
and result in inflammation.

338. Why are girls more prone to urinary tract infections than boys?
1. Girls have longer urethras and larger bladders
2. Girls tend to urinate less frequently
3. Girls are likely to have more bacteria due to the close proximity to the vagina
4. The propensity of bacterial attachment to the female periurethral mucosa and shorter urethras may
account for this difference
5. Only in girls the bacteria causing cystitis ascend to the kidney to cause pyelonephritis

339. Which of the following disease is a common complication of chronic constipation?


1. Pyelonephritis
2. Gastroenteritis
3. Pneumonia
4. Meningitis
5. Cellulitis

340. Which of the following is possible complication of vesicoureteral reflux?


1. Renal scarring
2. Renal stones
3. Cystitis
4. Renal vein thrombosis
5. Hypertension
341. Which hormone is a part of a hormone system that regulates blood pressure,
fluid and electrolyte balance, and systemic vascular resistance?
1. Angiotensin-II
2. Atrial natriuretic peptide
3. Parathyroid hormone (PTH)
4. 25OH Vitamin D
5. 3-Dehydroretinol

342. "A 3-month-old child, against the background of subfebrile body temperature and rhinitis, has pallor,
cyanosis of the nasolabial triangle, severe expiratory dyspnea, deflated chest, dry cough, participation of
auxiliary muscles in breathing. and small bubbling wet rales on both sides. In the blood: Hb - 112 g / l
a) acute bronchiolitis
b) acute (simple) bronchitis
c) bilateral pneumonia
d) bronchial asthma, exacerbation
e) acute obstructive bronchitis

343. "A 14-year-old boy complains of periodic abdominal pain that decreases after eating, belching,
weakness, black stools. Peptic ulcer is suspected. What study is the most informative in verifying the
diagnosis?"
A) X-ray
B) abdominal ultrasound
C) endoscopic
D) stool for occult blood
E) colonoscopy

344. Risk factors for the development of severe bronchiolitis:


1) The presence of respiratory syncytial infection
2) Female gender
3) Age more than 10 years
4) The presence of diseases of the musculoskeletal system
5) Atopic dermatitis

345.Which of the following is usually the first symptom of dilated cardiomyopathy?


A. Becoming short of breath during exertion
B. Fast heart rate
C. Fluid in the legs and abdomen
D. Flu like symptoms
E. Edema

You might also like